You are on page 1of 64

VISION IAS

www.visionias.in

Test Booklet Series

TEST-24
Time Allowed: Two Hours
GENERAL STUDIES (P) 2021 – Test – 3201
C
Maximum Marks: 200

INSTRUCTIONS

FULL(COMPLETE
LENGTH TEST
SYLLABUS)
FULL LENGTH TEST (COMPLETE SYLLABUS): Development +
Indian Constitution, Political System and Ecology & Environment
Governance + + General Science
Geography +
History of India + I

+ Current Affairs
1. With reference to characteristics of 4. The report “Assessment of Climate Change
'Persistent Organic Pollutants', consider the
over the Indian Region" has been released by
following statements:
1. They persist in the environment for long which of the following?

periods of time. (a) World Wide Fund for Nature-India


2. They are transported over large distances
(WWF-India)
through air and water.
3. They dissolve into the fatty tissues of (b) The Energy and Resources Institute
living organisms. (TERI)
Which of the statements given above is/are
(c) Ministry of Environment, Forest and
correct?
(a) 1 and 2 only Climate Change (MoEFCC)
(b) 2 only (d) Ministry of Earth Sciences (MoES)
(c) 1 and 3 only
(d) 1, 2 and 3
5. With reference to Sitanavasal paintings
2. With reference to Body Mass Index (BMI), considers the following statements:
consider the following statements:
1. These paintings are the rock-cut painting
1. It is a measure of the daily intake of
calorie-requirement by a person to found in the state of Karnataka.
sustain physiological activities. 2. These paintings are associated with
2. As per the World Health Organization
Jainism.
(WHO), the normal BMI ranges between
18.5 to 24.9. Which of the statements given above is/are
Which of the statements given above is/are correct?
correct?
(a) 1 only
(a) 1 only
(b) 2 only (b) 2 only
(c) Both 1 and 2 (c) Both 1 and 2
(d) Neither 1 nor 2
(d) Neither 1 nor 2

3. Which of the following were the immediate


outcomes of Bank Nationalization in 1969? 6. ‘Global Innovation and Technology Alliance
1. Increase in credit availability to the
(GITA)’, a platform encouraging industrial
agriculture sector.
2. Increase in financial savings. investments in innovative technology is
3. Increase in money supply leading to jointly developed by Department of Science
high inflation.
& Technology (DST) and
Select the correct answer using the code
given below. (a) National Informatics Centre
(a) 1 and 2 only (b) NITI Aayog
(b) 2 and 3 only
(c) Confederation of Indian Industry
(c) 1 and 3 only
(d) 1, 2 and 3 (d) Indian Science Congress

2 www.visionias.in ©Vision IAS


7. Consider the following statements regarding 10. In the context of Indian Depository Receipts,
the 'Birds of Prey': consider the following:
1. These are birds that are small in size and 1. It allows domestic investors to invest in
live together in large groups. the equity of companies of other foreign
2. Birds of prey have excellent vision with countries.
a wide binocular range. 2. An Indian Depository Receipt is
3. Sparrows and Parrots are examples of denominated in the Indian Rupee.
Birds of Prey. Which of the statements given above is/are
Which of the statements given above is/are correct?
(a) 1 only
correct?
(b) 2 only
(a) 2 only
(c) Both 1 and 2
(b) 2 and 3 only
(d) Neither 1 nor 2
(c) 2 only
(d) 1, 2 and 3
11. Consider the following statements about
trade winds:
8. Consider the following statements regarding
1. The trade winds are winds blow from
Parliamentary Standing Committees (PSC):
east to west just north and south of the
1. PSC are permanent bodies, constituted
equator.
every year and work in a continuous 2. Trade winds are more regular and
manner. constant over the lands than over oceans.
2. All bills have to be scrutinised by a Which of the statements given above is/are
Parliamentary Standing Committee. correct?
Which of the statements given above is/are (a) 1 only
correct? (b) 2 only
(a) 1 only (c) Both 1 and 2
(b) 2 only (d) Neither 1 nor 2
(c) Both 1 and 2
(d) Neither 1 nor 2 12. Consider the following statements about
Bimbisara:
9. In the context of Ramsar sites in India, 1. He was a contemporary of both Buddha
consider the following pairs: and Mahavira.
Ramsar Sites Located at 2. He pursued a three-pronged policy,
1. Kabartal Wetland : Uttarakhand namely, matrimonial alliances,
2. Asan Conservation : Uttar Pradesh friendship with strong rulers and
Reserves conquest of weak neighbours to expand
3. Nandur Madhameshwar the empire.
: Maharashtra
3. He made Patliputra as his capital.
bird sanctuary
Which of the statements given above is/are
Which of the pairs given above is/are
correct?
correctly matched?
(a) 1 and 3 only
(a) 1 only
(b) 1 and 2 only
(b) 1 and 2 only
(c) 2 and 3 only
(c) 2 and 3 only
(d) 3 only
(d) 3 only
3 www.visionias.in ©Vision IAS
13. In the context of developments resulting in 16. Consider the following statements with
the growth of militant nationalism, arrange reference to the metal 'Vanadium':
1. It is a high-value metal used in
the following events in chronological order.
strengthening steel.
1. Passage of Official Secrets Act
2. It is mostly recovered from the slag of
2. Deportation of Natu brothers iron-ore.
3. Indian Councils Act of 1892 3. In India, the largest reserves of
Select the correct answer using the code Vanadium can be found in Jharkhand.
Which of the statements given above is/are
given below.
correct?
(a) 2-1-3
(a) 1 and 2 only
(b) 3-2-1 (b) 2 only
(c) 1-2-3 (c) 1 and 3 only
(d) 1-3-2 (d) 1, 2 and 3

17. Consider the following pairs:


14. 'Polybezoar', a term recently seen in news is:
Regions sometimes State/UT
(a) A toxic chemical emitted from steel mentioned in news
factories. 1. Tawang : Arunachal Pradesh
(b) Hazardous pesticide recently banned by 2. Naku La : Sikkim
3. Demchok : Ladakh
the Government of India.
Which of the pairs given above are correctly
(c) A type of thermoplastic recently
matched?
developed to produce space equipment. (a) 1 and 3 only
(d) A tightly packed indigestible material (b) 1 only
that causes gastrointestinal blockages in (c) 2 and 3 only
(d) 1, 2 and 3
animals.

18. Consider the following statements about


15. In what ways can algal blooms prove Payment Banks:
harmful? 1. They are not required to meet the
1. Production of toxins Statutory Liquidity Ratio requirements
stipulated by RBI.
2. Creating anoxic conditions
2. They cannot issue loans or provide
3. Blocking light to organisms
credit card service.
4. Clogging of fish gills 3. They are not permitted to establish their
Select the correct answer using the code own ATMs.
given below. Which of the statements given above are not
correct?
(a) 1, 2 and 3 only
(a) 1 and 2 only
(b) 2 and 3 only
(b) 1 and 3 only
(c) 1 and 4 only (c) 2 and 3 only
(d) 1, 2, 3 and 4 (d) 1, 2 and 3

4 www.visionias.in ©Vision IAS


19. Amaravati Marbles, a collection of 120 22. Lord Lytton followed the policy of proud
sculptures and inscriptions, recovered from reserve against which of the following
regions?
the Amaravati Mahachaitya in Guntur and
(a) Nepal
currently housed in the British Museum in (b) Afghanistan
London are often associated with which of (c) Sind
the following archaeologists? (d) Madras
(a) Alexander Cunningham
23. Reserve Bank of India recently revised
(b) H. H. Cole
priority sector lending guidelines to include
(c) Walter Elliot entrepreneurship and renewable resources.
(d) James Prinsep In this context, consider the following
statements with reference to Priority Sector
Lending:
20. Which of the following are major areas of
1. The provision of PSL applies to
ravines and gullies in India? Regional Rural Banks, Local Area
1. Yamuna-Chambal zone Banks, and Urban Cooperative Banks.
2. The Punjab Siwalik foothills zone 2. All Regional Rural banks have to set
aside 40% of their adjusted net bank
3. Chhotanagpur zone
credit for PSL.
Select the correct answer using the code
3. Banks having a shortfall in PSL lending
given below. targets will be required to contribute to
(a) 2 and 3 only the Rural Infrastructure Development
(b) 1 and 2 only Fund.
Which of the statements given above is/are
(c) 1 and 3 only
correct?
(d) 1, 2 and 3 (a) 1 and 3 only
(b) 2 only
21. The Sufis of this order were known for their (c) 2 and 3 only
(d) 1, 2 and 3
close ties with the rulers and played a key
role in making war and peace. They acted as
24. Consider the following statements with
political emissaries and ambassadors and reference to Non-Banking Finance
held important posts as advisers in the royal Companies (NBFCs):
court. They accepted jagirs and gifts as royal 1. RBI regulates all the NBFCs in India.
2. Systemically important NBFCs are those
patronage. Although founded by Sheikh
whose asset size is ₹ 5000 crore or more.
Shihabuddin, it was established in India by 3. All NBFCs are required to maintain a
Sheikh Bahauddin Zakariya. liquidity buffer in terms of Liquidity
Which of the following Sufi order is being Coverage Ratio.
Which of the following statements given
described in the above passage?
above is/are correct?
(a) Qalandars (a) 1 and 2 only
(b) Naqshbandi (b) 3 only
(c) Chisti (c) 2 and 3 only
(d) Suhrawardi (d) None

5 www.visionias.in ©Vision IAS


25. Which of the following protected areas are 28. Sustainability in agriculture can also be
located in the Narmada river basin? increased through the role of legumes in the
1. Nauradehi Wildlife Sanctuary cropping system. Which of the following are
2. Melghat Tiger Reserve the advantages of legumes?
3. Guru Ghasidas National Park 1. It helps in nitrogen-fixation.
4. Tadoba National Park 2. It helps in solubilization of Phosphorous.
Select the correct answer using the code 3. It inhibits soil microbial activity.
given below. 4. It decreases mycorrhizal colonisation.
(a) 1, 2 and 3 only Select the correct answer using the code
(b) 2 and 4 only given below.
(c) 1, 3 and 4 only (a) 1 and 2 only
(d) 1, 2, 3 and 4 (b) 2 and 3 only
(c) 1, 3 and 4 only
26. PM CARES Fund recently seen in the news, (d) 1, 2, 3 and 4
consider the following statements:
1. The Minister of Home Affairs is the ex- 29. With regard to which of the following
officio chairman of Board of trustee of Mahatma Gandhi and Jawaharlal Nehru
fund. were in agreement?
2. It provides support and relief measure in 1. A technology-driven modern economy
case of natural as well as man-made 2. Personal religion
calamity or distress. 3. Parliamentary form of government
3. The fund is audited by the Comptroller Select the correct answer using the code
and Auditor General of India in every given below.
financial year, (a) 3 only
Which of the statements given above is/are (b) 1 and 2 only
correct? (c) 2 only
(a) 1 and 2 only (d) None
(b) 2 only
(c) 1 and 3 only 30. There is a divergence between the Wholesale
(d) 1, 2 and 3 Price Index (WPI) and Consumer Price
Index (CPI). What may be the possible

27. Which of the following is/are white blood causes for this divergence?

cells? 1. CPI basket consists of services such as

1. Neutrophils housing and education, which are not a

2. Lymphocytes part of the WPI basket.

3. Basophils 2. WPI does not include indirect taxes


while CPI includes it.
4. Erythrocytes
Which of the statements given above is/are
Select the correct answer using the code
correct?
given below.
(a) 1 only
(a) 1 and 3 only
(b) 2 only
(b) 2 only
(c) Both 1 and 2
(c) 1, 2 and 3 only
(d) Neither 1 nor 2
(d) 1, 2, 3 and 4
6 www.visionias.in ©Vision IAS
31. With reference to the associations that 33. In the context of medieval India, consider
preceded the Indian National Congress, the following statements regarding the Iqta
system:
consider the following pairs:
1. In India, iqta system was introduced by
Association Founder
Mahmud of Ghazni
1. Madras Mahajan : P Ananda
2. Iqtas were granted only for
Sabha Charlu administrative and revenue purposes.
2. Indian Association : K. T. Telang Which of the statements given above is/are
3. Bombay Presidency : Dinshaw Petit correct?

Association (a) 1 only


(b) 2 only
Which of the pairs given above is/are
(c) Both 1 and 2
correctly matched?
(d) Neither 1 nor 2
(a) 3 only
(b) 1 and 2 only 34. With reference to Aerial seeding, consider
(c) 1, 2 and 3 the following statements:
(d) 1 only 1. It is a technique of plantation in which
seeds covered with clay and compost are
sprayed on the ground.
32. Consider the following statements with
2. It is only suitable for regions with heavy
respect to the Essential Commodities Act
and regular precipitation.
1955: Which of the statements given above are
1. The Act empowers the governments to correct?
control the licensing and distribution of (a) 1 only
(b) 2 only
certain commodities in view of rising
(c) Both 1 and 2
prices.
(d) Neither 1 nor 2
2. Under the act, the government cannot fix
the price limits of any commodity. 35. Consider the following statements:
3. Recently, government-imposed 1. Front is not formed at the equator.
Minimum Export Price on Onion under 2. In divergent circulation no front is

the provisions of the Essential formed.


3. Cold front is associated with thick
Commodities Act 1955.
clouds and heavy downpour.
Which of the statements given above is/are
Which of the statements given above are
correct? correct?
(a) 1 only (a) 1 and 2 only
(b) 1 and 2 only (b) 2 and 3 only

(c) 1 and 3 only (c) 1 and 3 only


(d) 1, 2 and 3
(d) 2 and 3 only

7 www.visionias.in ©Vision IAS


36. With reference to the state-level Death/ 39. Consider the following statements, with
Mortality Audit Committee', consider the respect to Non-Aligned Movement (NAM):
following statements: 1. The Bandung Conference led to the
1. It is set up by the Indian Council of the creation of NAM.
2. India and Pakistan are its founding
Medical Research.
members.
2. Its members are nominated by the State
3. Currently, India holds the presidency of
governments.
the summit till 2022.
Which of the statements given above is/are
Which of the statements given above are
correct? correct?
(a) 1 only (a) 1 only
(b) 2 only (b) 1 and 2 only
(c) Both 1 and 2 (c) 2 and 3 only
(d) Neither 1 nor 2 (d) 1, 2 and 3

37. With reference to modern Indian history, 40. Which of the following can be stated as the

Muddiman committee, led by Alexander advantages of Spot exchanges?


1. Prevent cartelization
Muddiman was appointed to
2. Avoidance of counterparty risk
(a) consider the adjustments desirable inside
3. Effective price determination
the framework of the Act of 1919.
Select the correct answer using the code
(b) investigate into the Jallianwala Bagh
given below.
massacre. (a) 1, 2 and 3
(c) decide on the reforms related university- (b) 1 and 2 only
level education in India. (c) 2 and 3 only
(d) None of the above (d) 3 only

38. Which of the following taxes given below 41. With reference to the Cicadas, consider the

are subsumed under Goods and Services Tax following statements:


1. They are protozoans which cause
(GST)?
diseases in humans and animals.
1. Countervailing duties
2. They spend most of their lives
2. Central Excise duty
underground and emerge from the soil
3. Basic customs duty
mainly to mate.
4. Luxury Tax 3. They are endemic to the Western Ghats
5. Stamp duties in India.
Select the correct answer using the code Which of the statements given above is/are
given below. correct?
(a) 4 and 5 only (a) 1 only
(b) 2, 3 and 4 only (b) 2 only
(c) 1, 2 and 3 only (c) 2 and 3 only
(d) 1, 2 and 3
(d) 1, 2 and 4 only

8 www.visionias.in ©Vision IAS


42. The word ‘Ransomware’ is sometimes 44. Which of the following is/are the sources of
mentioned in news, which of the following Styrene Gas?
1. Vehicle exhaust
statements best describes it ?
2. Cigarette smoke
(a) It denotes to the cheating an internet user 3. During the production of copolymers
through a fake SMS or email and 4. During rocket propulsion
Select the correct answer using the code
thereby breaching their privacy to steal
given below.
sensitive information.
(a) 1 and 2 only
(b) It refers to the malicious software that (b) 1, 2 and 3 only
locks up the files on your computer in (c) 2, 3 and 4 only
(d) 1, 2, 3 and 4
encrypted form and demands a pay to
get your files back. 45. Consider the following statements regarding
(c) It refers to the attack which overwhelm Coccolithophores:
the computers with more traffic than the 1. They are single-celled algae living in the
upper layers of the world's oceans.
server or network can accommodate.
2. They calcify marine phytoplankton to
(d) It refers to the eavesdropping attacks, produce open ocean calcium carbonate.
occurs when attackers insert themselves Which of the statements given above is/are

into a two-party transaction and steal the correct?


(a) 1 only
data.
(b) 2 only
(c) Both 1 and 2
43. Which of the following can be considered as (d) Neither 1 nor 2

discretionary powers of the Governor?


46. In the context of modern Indian history,
1. Reservation of a bill for the consider the following statements with
consideration of the President. reference to provincial elections held in
2. Pardoning powers of the Governor. 1937:
1. Provincial elections of 1937 were
3. Recommendation for the imposition of
conducted under the provisions of the
the President’s Rule in the state. Government of India Act 1935.
4. Dissolution of the state legislative 2. Muslim League was the single largest

assembly if the council of ministers party in Bengal province.


3. Mahatma Gandhi commended the
loses majority.
working of Provincial ministries for their
Select the correct answer using the code corruption-free governance.
given below. Which of the statements given above is/are
correct?
(a) 1 and 3 only
(a) 1 and 2 only
(b) 2, 3 and 4 only (b) 1 only
(c) 1, 3 and 4 only (c) 2 and 3 only
(d) 1, 2 and 4 only (d) 1, 2 and 3

9 www.visionias.in ©Vision IAS


47. Consider the following statements with 50. Which of the following events occur during
reference to the 'Scorpene-Class
the process of photosynthesis?
Submarines':
1. They are diesel-electric powered attack 1. Conversion of chemical energy into light
submarine. energy.
2. They are being constructed with
2. Splitting of water molecules into
technology transfer from Russia.
3. They are named after the different hydrogen and oxygen.
islands of the Lakshadweep.
3. Reduction of oxygen to carbohydrates.
Which of the statements given above
is/are correct? Select the correct answer using the code
(a) 1 only given below.
(b) 1 and 2 only
(a) 1, 2 and 3
(c) 2 and 3 only
(d) 1 and 3 only (b) 2 only

(c) 1 and 2 only


48. Which of the following is/are classified as
components of Financial Market (d) 1 and 3 only
Infrastructure (FMI) in India?
1. Real-Time Gross Settlement System
51. It is created for the administration of a fast
(RTGS)
2. Central Depository Services Ltd. developing town due to industrialisation,
(CDSL)
and a town which does not yet fulfil all the
3. Clearing Corporation of India Ltd
(CCIL) conditions necessary for the constitution of a
Select the correct answer using the code municipality. Though it functions within the
given below.
framework of the State Municipal Act, only
(a) 1 and 2 only
(b) 2 and 3 only certain provisions of the act apply to it.
(c) 1 only
Unlike the municipality, it is an entirely
(d) 1, 2 and 3
nominated body, that is, all the members
49. Which of the following events are associated including the chairman are nominated by the
with the Civil Disobedience Movement?
state government.
1. Refusal to pay the chowkidar taxes
2. Implementation of Cunningham Circular The above description refers to which of the
3. Individual Satyagraha
following types of urban local bodies?
Select the correct answer using the code
given below. (a) Town Area Committee
(a) 1 and 2 only (b) Notified Area Committee
(b) 2 and 3 only
(c) Municipal Council
(c) 1 and 3 only
(d) 1, 2 and 3 (d) Cantonment Board

10 www.visionias.in ©Vision IAS


52. Which of the following statements is/are 55. Consider the following statement regarding
correct with reference to Ordinance making the differences between the World Bank and
power of the President?
the International Monetary Fund (IMF):
1. The President can promulgate an
ordinance even when one House is in 1. While the World Bank aims at the
session. development of poor countries, the IMF
2. The decision of the President to issue an is responsible for maintaining a system
ordinance cannot be questioned in any
of payments and receipts between
Court.
3. Ordinance power of the President cannot nations.
be used to amend the Constitution. 2. While the IMF lends specifically for
Select the correct answer using the code development projects, the world bank
given below.
aims to address the overall financial
(a) 1 and 2 only
(b) 2 and 3 only needs of a country.
(c) 3 only 3. While the IMF can only lend to national
(d) 1 and 3 only governments, the world bank can also
lend to individuals.
53. Insects provide ecological services by
Which of the statements given above is/are
playing which of the following roles?
1. Dispersal agents correct?
2. Food source (a) 1 and 2 only
3. Pollinators
(b) 1 only
4. Decomposers
(c) 2 and 3 only
Select the correct answer using the code
given below. (d) 1, 2 and 3
(a) 1 and 3 only
(b) 2 and 3 only
56. Consider the following statements regarding
(c) 1,3 and 4 only
Torpor, a state of survival in animals:
(d) 1, 2, 3 and 4
1. To enter torpor an animal decreases its
54. Consider the following statements with metabolism and reduces its energy
reference to the Second Roundtable requirements.
Conference:
2. It is only observed in birds and
1. It was the only Roundtable Conference
in which Gandhiji participated. mammals of the northern hemisphere.
2. The Civil Disobedience Movement 3. Some animals can enter torpor daily for
which started in 1930 came to an end a few hours.
after Gandhi's return from the
Which of the statements given above is/are
conference in London.
Which of the statements given above is/are correct?
correct? (a) 1 and 3 only
(a) 1 only (b) 2 and 3 only
(b) 2 only
(c) 1 and 2 only
(c) Both 1 and 2
(d) Neither 1 nor 2 (d) 1, 2 and 3

11 www.visionias.in ©Vision IAS


57. In the context of Indian economy, which of 60. Which of the following statements is/are
correct with reference to the state of
the following five-year plans proposed to
agriculture during the Mughal period?
achieve the two main objectives of 'removal 1. Peasants were encouraged to grow both
of poverty' (Garibi Hatao) and 'attainment of subsistence and commercial crops.
2. Irrigation never received any support
self-reliance'?
from the state.
(a) First five-year plan 3. Non-resident cultivators of a village
(b) Second five-year plan were allowed to cultivate lands on a
contractual basis.
(c) Fourth five-year plan
Select the correct answer using the code
(d) Fifth five-year plan given below.
(a) 1 and 2 only
(b) 1 and 3 only
58. With reference to the Indian Pharmaceutical
(c) 3 only
Industry, which of the following statements (d) 2 and 3 only
is/are correct?
61. Consider the following statements with
1. India's Pharma Industry is the world's
reference to vertical farming:
third-largest by value. 1. It is the practice of producing food in
vertically stacked layers in an outdoor
2. 100% FDI is allowed under automatic
environment using only natural sunlight.
route for Greenfield Pharma. 2. It is being progressively practised in the
3. Himachal Pradesh has Asia’s largest production of poultry and leafy-
vegetables.
pharma manufacturing hub.
Which of the statements given above is/are
Select the correct answer using the code correct?
given below. (a) 1 only
(b) 2 only
(a) 1 and 2 only
(c) Both 1 and 2
(b) 2 only (d) Neither 1 nor 2
(c) 2 and 3 only
62. Which of the following committees made
(d) 1 and 3 only
recommendations with respect to Centre-
State relations?
59. 'Pantanal' the world's largest tropical wetland 1. Sarkaria Commission
2. M M Punchhi Commission
recently seen in the news, spreads across
3. Rajamannar Committee
which of the following countries? Select the correct answer using the code
given below.
(a) Kenya, Uganda and Tanzania
(a) 1 and 2 only
(b) USA and Canada (b) 2 and 3 only
(c) Thailand, Laos and Cambodia (c) 1 and 3 only
(d) 1, 2 and 3
(d) Bolivia, Brazil and Paraguay
12 www.visionias.in ©Vision IAS
63. Which one of the following Fundamental 66. Which of the following statements regarding
Rights incorporates the Right to Elementary Judicial activism in India is/are correct?

Education? 1. It is defined in the constitution of India.


2. It has strengthened the doctrine of
(a) The Cultural and Educational Rights
separation of powers.
(b) Right to Freedom
3. It has enforced executive responsibility.
(c) Right to Equality Select the correct answer using the code
(d) Right to Constitutional Remedies given below.
(a) 1 only

64. Aarogya Setu App is often in news, with (b) 3 only


(c) 1 and 2 only
respect to it, consider the following
(d) 2 and 3 only
statements:
1. It is a contact tracing app in order to
67. With reference to India's External Debt,
inform the app user about his/her consider the following statements:
interaction with COVID-19 positive 1. Within India's overall external debt,
patient. sovereign debt is twice as large as the

2. It is mandatory for air and rail travel. private sector debt.


2. Outstanding NRI deposits are the largest
Which of the statements given above is/are
constituent of India's external debt.
correct?
3. Majority of India's external debt is
(a) 1 only denominated in the Chinese Renminbi.
(b) 2 only Which of the statements given above is/are
(c) Both 1 and 2 correct?

(d) Neither 1 nor 2 (a) 1 only


(b) 2 and 3 only
(c) 1 and 2 only
65. "The crop requires a humid climate with
(d) None
temperature fluctuating between 24 to 38
degree Celsius. Minimum rainfall required 68. Consider the following diseases:
for its cultivation is 1000 mm. The new grey 1. Hepatitis A
alluvial soil of good depth receiving silt 2. Hepatitis B

from annual floods is most suitable for the 3. Human Immunodeficiency Virus (HIV)
4. syphilis
growth of the crop."
Which of the given above are blood-borne
Which one of the following is that crop?
diseases?
(a) Cotton (a) 1 and 2 only
(b) Wheat (b) 2, 3 and 4 only
(c) Jute (c) 1 and 3 only
(d) Sugarcane (d) 1, 2, 3 and 4

13 www.visionias.in ©Vision IAS


69. With reference to Viability Gap Funding 72. The Global Wage Report 2020-21, is
Scheme, consider the following statements: released by
1. The funding is provided by both the (a) International Labour Organization
Central and state governments. (b) World Bank
2. This scheme is provided for the (c) World Economic Forum
completion of the social sector projects.
(d) International Monetary Fund
Which of the statements given above is/are
correct?
73. "They are created when giant stars die in
(a) 1 only
supernovas and their cores collapse. They
(b) 2 only
pack their mass inside a 20-kilometer
(c) Both 1 and 2
(d) Neither 1 nor 2 diameter. They are so dense that a single
teaspoon would weigh a billion tons. On

70. Indian Penal Code often seen in the news average, gravity on this star is 2 billion times
contains provisions related to stronger than gravity on Earth." The star in
1. Prescription of penalties and the the above passage refers to
punishment for crimes (a) Quasar
2. Procedure for crime investigation (b) White Dwarf
3. Powers of the Courts and Magistrate (c) Neutron
Select the correct answer using the code (d) Red Giant Stars
given below.
(a) 1 only
74. Consider the following statements with
(b) 2 and 3 only
reference to Urea subsidy in India:
(c) 1 and 3 only
1. Currently, the Government gives
(d) 1, 2 and 3
uniform subsidy on per nutrient basis to
all urea manufacturers.
71. Consider the following statements in the
context of the Director of the Central Bureau 2. Direct cash transfer (DCT) in urea

of Investigation (CBI): subsidy could curb overuse of urea by


1. He is appointed by the recommendation farmers.
of a five-member committee headed by 3. Irrational use of urea owing to high
the Prime Minister. subsidy has been linked with a higher
2. Only Central Vigilance Commissioner incidence of blue baby syndrome in
has the authority to remove or suspend India.
the Director of CBI. Which of the statements given above are
Which of the statements given above is/are correct?
correct?
(a) 1 and 2 only
(a) 1 only
(b) 2 and 3 only
(b) 2 only
(c) 1 and 3 only
(c) Both 1 and 2
(d) 1, 2 and 3
(d) Neither 1 nor 2
14 www.visionias.in ©Vision IAS
75. A person who is arrested or detained under 77. In the context of offices and posts prevalent
during the various periods of the ancient
ordinary law (punitive detention) is entitled
Indian history, which of the following pairs
to which of the following rights under the is/are correctly matched?
constitution? Term Meaning/Function
1. Sandhivigrahika : Minister of peace
1. The right to be released after one month
and war
unless the magistrate authorises further
2. Pradvivaka : Chief record keeper
detention. and secretary

2. Right to consult and be defended by a 3. Mahaksapatalika : Chief tax-collector


Select the correct answer using the code
legal practitioner.
given below.
3. Right to be informed of the grounds of (a) 1 only
(b) 1 and 3 only
arrest.
(c) 2 and 3 only
Select the correct answer using the code (d) 1, 2 and 3
given below.
78. Consider the following statements with
(a) 1 only reference to the 'End-to-End Encryption'
(b) 2 and 3 only communication system:
1. The message travels from the sender to
(c) 1 and 3 only
the server as plain text, the server
(d) 1, 2 and 3 encrypts and transfers it to the recipient
2. The data sent over Short Messaging
Services are not end-to-end encrypted
76. In the context of ancient Indian philosophy, Which of the statements given above is/are
consider the following statements: correct?
(a) 1 only
1. Both Ajivika and Ajnana sects belonged (b) 2 only
to the Shramana tradition. (c) Both 1 and 2
(d) Neither 1 nor 2
2. Ajivika was an atheist sect that believed

in Karma doctrine, unlike Jainism and 79. With reference to Attorney General of India,
consider the following statements :
Buddhism.
1. He should be qualified to be appointed
3. Ajnana sect believed that Knowledge as Judge of Supreme Court.
about nature is useless to attain 2. He holds office at the pleasure of the
President of India.
salvation.
3. He has the right to attend, speak and
Which of the statements given above are vote in proceedings of both houses of
parliament.
correct?
Which of the statements given above is/are
(a) 1 and 2 only correct?
(b) 2 and 3 only (a) 1 and 3 only
(b) 1 only
(c) 1 and 3 only (c) 1 and 2 only
(d) 1, 2 and 3 (d) 3 only
15 www.visionias.in ©Vision IAS
80. Consider the following statements with 83. Which of the following statements is/are
respect to the Global Atmosphere Watch: correct regarding the office of the Cabinet
1. It was established by the United Nations Secretary?
Environment Programme. 1. She/He is nominated by the President of
2. It aims to understand the interactions India.
between the atmosphere, the oceans and 2. She/He is the ex-officio Chairman of the
the biosphere. Civil Services Board.
3. Every year it declares Earth Overshoot 3. She/He presides over the Annual
Day. conference of Chief Secretaries.
Which of the statements given above is/are
Select the correct answer using the code
correct?
given below.
(a) 1 and 2 only
(a) 1 only
(b) 3 only
(b) 1 and 2 only
(c) 1 and 3 only
(c) 2 only
(d) 1, 2 and 3
(d) 2 and 3 only

81. Consider the following statements about Sir


84. Some economic indicators post lifting of
Syed Ahmed Khan:
1. He preached obedience to British rule. lockdown point towards a strange
2. He opposed imparting modern scientific phenomenon where the unemployment rate
education for Muslims which shot up immediately after the
3. He opposed the formation of the Indian lockdown went back to pre-Covid levels but
National Congress. leaving India with 20-40 million fewer
Which of the statements given above is/are employed people than it had before Covid.
correct? This is due to:
(a) 1 and 3 only (a) Low levels of Unemployment Rate and
(b) 2 and 3 only high levels of Worker Population Ratio
(c) 2 only (b) High levels of Unemployment Rate and
(d) 1, 2 and 3 low levels of Worker Population Ratio
(c) Low levels of Unemployment Rate and
82. Consider the following statements, with Low levels of Worker Population Ratio
respect to the ARYA Project: (d) High levels of Unemployment Rate and
1. It provides scholarship to youths who High levels of Worker Population Ratio
are doing research in the field of
agriculture. 85. Consider the following pairs:
2. It mandates that at least one-third
Textile craft Heritage of
beneficiaries should be women.
1. Bagh print : Bihar
3. It is implemented by Krishi Vigyan
2. Tanchoi style : Manipur
Kendras in atleast one district from each
3. Bagru style : Rajasthan
state.
Which of the pairs given above is/are
Which of the statements given above is/are
correctly matched?
correct?
(a) 1 and 3 only
(a) 1 and 2 only
(b) 1 and 2 only
(b) 2 and 3 only
(c) 3 only (c) 3 only
(d) 1, 2 and 3 (d) 1, 2 and 3

16 www.visionias.in ©Vision IAS


86. Which of the following is/are the reasons for 89. Consider the following statements:
selecting Thoothukudi as India's second 1. Public Debt in India comprises only the
launch pad?
market borrowings of the government.
1. It's geographical location helps avoid
2. At present, Public debt in India is
deviation from a straight flight path.
managed by the Public Debt
2. Proximity to Earth’s equator as
compared to the first launchpad. Management Agency under the Ministry
3. Proximity to critical space infrastructure. of Finance.
Select the correct answer using the code Which of the statements given above is/are
given below. correct?
(a) 1 only
(a) 1 only
(b) 1 and 2 only
(b) 2 only
(c) 2 and 3 only
(d) 1, 2 and 3 (c) Both 1 and 2
(d) Neither 1 nor 2
87. Consider the following statements with
reference to Permanent Lok Adalats: 90. The term 'Medicanes' recently seen in the
1. Only a person who is or has been a
news refers to
District judge can become the chairman
(a) Cyclone observed across the
of the permanent Lok Adalat.
2. Permanent Lok Adalats have been given Mediterranean Sea.
statutory status under the Legal Services (b) A short-lived change in the color and
Authorities Act, 1987. appearance of the Moon.
Which of the statements given above is/ are (c) A genetically modified sugarcane
correct?
variety which can be consumed by
(a) 1 only
diabetic patients.
(b) 2 only
(c) Both 1 and 2 (d) An app by the Indian Government to
(d) Neither 1 nor 2 track the COVID vaccination
programme.
88. Which of the following are examples of
commensalism?
91. The term 'BreathPrint', recently seen in the
1. Mites and Humans
news is related to:
2. Epiphytic plants
3. Hermit crabs and Sea Anemones (a) A method to detect the CoVID-19.

4. Cattle and Egrets (b) A device to detect the level of alcohol in


Select the correct answer using the code the blood.
given below. (c) A method to detect the ulcer-causing
(a) 1, 2 and 3 only
gastric pathogen.
(b) 3 and 4 only
(d) A device to control the level of
(c) 1, 2 and 4 only
(d) 1, 2, 3 and 4 pollution.

17 www.visionias.in ©Vision IAS


92. Consider the following pairs: 94. Formed in the year 1920, All India Trade
Union Congress (AITUC), has completed
Folk dance State of origin 100 years of existence. In this context,
1. Hojagiri : Andhra Pradesh consider the following statements:
1. It is widely regarded as the first
2. Cheraw : Mizoram organised trade union in India.
2. It was formed to give India
3. Kamsale : Karnataka
representation at the International
Which of the pairs given above is/are Labour Organization (ILO) of the
League of Nations
correctly matched? 3. Appreciating the union as the need of the
hour, Mahatma Gandhi attended its first
(a) 2 only
session.
(b) 1 and 3 only Which of the statements given above is/are
correct?
(c) 2 and 3 only (a) 1 only
(d) 1, 2 and 3 (b) 1 and 2 only
(c) 1, 2 and 3
(d) 2 only

93. With reference to the Nominated Members


95. Consider the following statements:
of Rajya Sabha, consider the following 1. No criminal proceedings can be
instituted or continued against the
statements: President or the Governor in any court
during his term of office.
1. Nominated members can vote in the
2. No process for the arrest or
election of the President and Vice- imprisonment of the President or the
Governor shall be issued from any court
President of India. during his term of office.
2. Within six months nominated members Which of the statements given above is/are
correct?
can join a political party without any (a) 1 only
(b) 2 only
disqualification.
(c) Both 1 and 2
3. Nominated members of either house of (d) Neither 1 nor 2

the Parliament take part in the 96. Consider the following statements:
1. The word Cabinet is no where
impeachment of the President.
mentioned in the Constitution.
Which of the statements given above are 2. Union Cabinet is the supreme executive
authority of Central government.
correct? 3. The decisions of the Union Cabinet are
(a) 1 and 3 only binding on the Council of Ministers.
Which of the statements given above are
(b) 2 and 3 only correct?
(a) 1 and 2 only
(c) 1 and 2 only
(b) 2 and 3 only
(d) 1, 2 and 3 (c) 1 and 3 only
(d) 1, 2 and 3
18 www.visionias.in ©Vision IAS
97. With reference to embryo twinning, consider 99. Consider the following statements related to
the following statements: Mahatma Gandhi National Rural
1. It refers to the formation of twins Employment Guarantee Act 2005
through the artificial microsurgical (MGNREGA):

twinning of an embryo at the blastocyst 1. It provides at least 100 days of


guaranteed wage employment in a
stage.
financial year, to every household whose
2. Embryo twinning increases the chance
adult members volunteer to do unskilled
of successful implantation and therefore
manual work.
the overall likelihood of a pregnancy.
2. The employment under the act is to be
Which of the statements given above is/are
provided within 15 km of an applicant's
correct? residence.
(a) 1 only 3. As per the act, applicants are entitled to
(b) 2 only an unemployment allowance if work is
(c) Both 1 and 2 not provided within 15 days of
(d) Neither 1 nor 2 applying.
Which of the statements given above is/are
98. The Constitution empowers the Parliament correct?
to make laws on any matter enumerated in (a) 2 only

the State List under which of the following (b) 1 and 2 only
(c) 1, 2 and 3
circumstances?
(d) 1 and 3 only
1. While a proclamation of national
emergency is in operation.
100. Consider the following statements regarding
2. When a President’s rule is imposed in a
the Kaziranga National Park:
state.
1. Brahmaputra River forms southern and
3. For implementing the international the Dihang river forms its northern
treaties, agreements or conventions boundary
4. When a State Legislature passes 2. It has the largest population of the Wild
resolutions requesting the Parliament to Water Buffalo.
enact laws on a matter in the State List. 3. India’s only Golden Tiger resides here.
Select the correct answer using the code Which of the statements given above is/are
given below. not correct?
(a) 1, 2 and 3 only (a) 1 only

(b) 1, 3 and 4 only (b) 2 and 3 only


(c) 1 and 2 only
(c) 2 and 4 only
(d) 1, 2 and 3
(d) 1, 2, 3 and 4

Copyright © by Vision IAS


All rights are reserved. No part of this document may be reproduced, stored in a retrieval system or transmitted
in any form or by any means, electronic, mechanical, photocopying, recording or otherwise, without prior
permission of Vision IAS.
19 www.visionias.in ©Vision IAS
VISIONIAS
www.visionias.in
ANSWERS & EXPLANATIONS
GENERAL STUDIES (P) TEST – 3201 (2021)

Q 1.D
 Bioaccumulation and biomagnification are two different processes that often occur in tandem with one
another. Bioaccumulation is the process by which toxins enter the food web by building up in individual
organisms, while biomagnification is the process by which toxins are passed from one trophic level to the
next (and thereby increase in concentration) within a food web.
 Synthetic (man-made) chemicals called Persistent Organic Pollutants, or POPs, are of primary
concern when looking at bioaccumulation and biomagnification.
 These chemicals do not easily break down in the environment and can build up in the fatty tissues of
living organisms. Some examples of POPs you may have heard of include DDT (an insecticide that was
used extensively post-WWII) and PCBs (flame retardants).
 Although the production of these chemicals was banned during the 1970s and 1980s, they can still be
found in the oceans as well as the tissues of many marine animals because of their ability to (1.) persist in
the environment for long periods of time, (2.) move within the water (they are transported over
large distances through air and water), and (3.) dissolve into the fatty tissues of living
organisms. For all of these reasons, POPs like DDT and PCBs are especially good at bioaccumulating
and biomagnifying. Hence the correct answer is option (d).
 Bioaccumulation occurs at the base of a food web, usually within primary producers like
phytoplankton. These microscopic photosynthetic organisms absorb POPs directly from the seawater and
accumulate them in their bodies over time. The toxins build up in their tissues because they are absorbed
from the water at a rate faster than they can be metabolized.
 Biomagnification occurs when slightly larger organisms called zooplankton feed upon the
contaminated phytoplankton and in turn absorb POPs into their own tissues at a higher
concentration. The more contaminated phytoplankton zooplankton eats, the more pollutants it will have
in its body. In other words, the POPs can be passed from producer to consumer (to consumer, to
consumer, and so on.)
 Biomagnification can continue all the way up the food web or chain. Because the amounts of POPs
become more and more concentrated at each trophic level, some of the ocean‘s apex predators are at risk
of gaining potentially fatal levels of POPs within their bodies.

Q 2.B
 The Body Mass Index (BMI) is measured as the weight in kg divided by the square of the height in
meters. Thus It is a measure of body fat based on height and weight. Hence statement 1 is not
correct.
 The WHO guidelines define a normal BMI range as 18.5 to 24.9, overweight as 25 or higher, and
obesity as 25 or higher. Hence statement 2 is correct.
 According to a recent study published in The Lancet, India ranks third (198th) and fifth (196th)
from the bottom respectively among countries where 19-year-old girls and boys have a low body mass
index.
 The mean BMI of 19-year-old boys is 20.1 in India, compared to a high of 29.6 in the Cook Islands and
a low of 19.2 in Ethiopia. For Indian girls, the mean BMI is again 20.1, compared to a high of 29.0 in
Tonga and a low of 19.6 in Timor-Leste. The mean height of Indian 19-year-olds is 166.5 cm for boys and
155.2 cm for girls, well below the high of Netherlands boys (183.8 cm) and girls (170 cm).

1 www.visionias.in ©Vision IAS


Q 3.D
 The nationalization of 14 major commercial banks was announced by the government of India through an
ordinance under Art. 123 of the constitution of India on July 19, 1969, which was later replaced with the
Banking Companies Act (Acquisition and transfer of undertakings) passed by the parliament on August 9,
1969.
 The fourteen (14) major commercial banks with a deposit of Rs. 50 crores or above were nationalized by
the govt, of India under the Banking Companies Act 1969. Reserve Bank of India describes bank
nationalization as the single-most-important economic policy decision taken by any government after
1947.
 The main cause of nationalization of banks has been stated as to include the enlargement of resources for
economic growth, the development of agriculture and Industry in backward regions as well as making
bank credit available to priority areas which had generally been neglected.
 The major outcomes of the banking industry in the first-decade post nationalization (1970-80) includes:
o Tremendous improvement in branch expansion especially in rural areas. The number of rural bank
branches increased ten-fold from about 1,443 in 1969 to 15,105 in 1980.
o Between 1969 and 1980, credit to agriculture expanded forty-fold from 67 crores to 2,767
crores. Hence, statement 1 is correct.
o Financial savings rose as lenders opened new branches in areas that were unbanked. Gross domestic
savings almost doubled as a percentage of national income in the 1970s. Hence, statement 2 is
correct.
o The inter-state disparity in the availability of bank offices declined.
o The growth of the money supply which had been more or less a single-digit figure in the previous
decade suddenly increased to 10.5 per cent in 1969-70. Between 1961 and 1970, the money supply
went up 2.2 times certainly given a fillip to prices and inflation. Hence, statement 3 is correct.

Q 4.D
 Assessment of climate change over the Indian region:
o The first ‗Assessment of Climate Change over Indian Region‘ was recently released by the Union
Ministry of Earth Sciences. Hence option (d) is the correct answer.
 Temperature Rise Over India
o India‘s average temperature has risen by around 0.7°C during 1901–2018. This rise in temperature is
largely on account of GHG-induced warming, partially offset by forcing due to anthropogenic
aerosols and changes in Land use land cover (LULC)
 Indian Ocean Warming
o Sea surface temperature (SST) of the tropical Indian Ocean has risen by 1°C on average during 1951–
2015, markedly higher than the global average SST warming of 0.7°C, over the same period.
 Changes in Rainfall
o The summer monsoon precipitation (June to September) over India has declined by around 6% from
1951 to 2015, with notable decreases over the Indo-Gangetic Plains and the Western Ghats.
o The frequency of localized heavy precipitation occurrences has increased worldwide in response to
increased atmospheric moisture content. Over central India, the frequency of daily precipitation
extremes with rainfall intensities exceeding 150 mm per day increased by about 75% during 1950–
2015.
 Droughts
o The overall decrease of seasonal summer monsoon rainfall during the last 6–7 decades has led to an
increased propensity for droughts over India. Both the frequency and spatial extent of droughts have
increased significantly during 1951–2016. In particular, areas over central India, southwest coast,
southern peninsula and north-eastern India have experienced more than 2 droughts per decade, on
average, during this period.
 Sea Level Rise
o Sea levels have risen globally because of the continental ice melt and thermal expansion of ocean
water in response to global warming. Sea-level rise in the North Indian Ocean (NIO) occurred at a
rate of 1.06–1.75 mm per year during 1874–2004 and has accelerated to 3.3 mm per year in the last
two and a half decades (1993–2017), which is comparable to the current rate of global mean sea-level
rise.
 Tropical Cyclones
o There has been a significant reduction in the annual frequency of tropical cyclones over the NIO basin
since the middle of the twentieth century (1951–2018). In contrast, the frequency of very severe

2 www.visionias.in ©Vision IAS


cyclonic storms (VSCSs) during the post-monsoon season has increased significantly (+1 event per
decade) during the last two decades (2000–2018).
 Changes in the Himalayas
o The Hindu Kush Himalayas (HKH) experienced a temperature rise of about 1.3°C during 1951–2014.
Several areas of HKH have experienced a declining trend in snowfall and also retreat of glaciers in
recent decades. In contrast, the high-elevation Karakoram Himalayas have experienced higher winter
snowfall that has shielded the region from glacier shrinkage.
 By the end of the twenty-first century, the annual mean surface temperature over HKH is projected to
increase by about 5.2°C under the RCP8.5 scenario.

Q 5.B
 Sittanavasal Cave also known as Arivar Koil is a Jain complex of caves in the Sittanavasal village of
Tamil Nadu. Hence statement 1 is not correct.
 These are examples of rock-cut architecture based on Jain thoughts and ideologies. Hence statement 2
is correct.
 They have a close form of Ajanta and Bagh caves. The name Sittanavasal has diverse cultural
connotations.
 Most paintings date to the Pandyan period i.e. 9th century AD.
 The notable drawings include a pond with lotuses. This scene shows Samava-sarana –a unique, chosen
and elegant audience hall where Tirthankaras gave sermons after they reached realisation (kevalya-jnana).
Bulls, elephants, apsaras (angels) and other Gods gathered in this audience hall to witness this grand
scene.

Q 6.C
 Recent Context: In November, the Union Minister of Science & Technology, Health and Family Welfare
and Earth Sciences, has underlined how Global Innovation and Technology Alliance (GITA) served as a
catalyst for nurturing innovation and industrial R&D by fostering bilateral academic industry and
government collaborations, through a video message at celebration of the 9th Foundation Day of GITA.
 Global Innovation & Technology Alliance (GITA) is a ―not–for–profit‖ Section-8 Public-Private
Partnership (PPP) company promoted jointly by the Technology Development Board (TDB),
Department of Science & Technology (DST), Government of India and the Confederation of Indian
Industry (CII). Hence option (c) is the correct answer.
 The Prime Minister‘s Council on Trade & Industry had, in 2010, recommended incorporation of a
Government‘s arm‘s length entity under PPP mode to professionally manage the Government‘s funds for
providing flexibility to industry for undertaking R&D along with global partners.
 The GITA platform encourages industrial investments in innovative technology solutions by:
o Mapping technology gaps,
o Undertaking expert evaluation of technologies available across the globe,
o Facilitating techno–strategic collaborative partnerships appropriate for the Indian economy
o Connecting industrial and institutional partners for synergistic matchmaking and
o Providing soft funding for technology development/acquisition/deployment.

Q 7.A
 Birds of prey, also known as raptors, include species of bird that primarily hunt and feed on
vertebrates that are large relative to the hunter. Millions of birds live all over the world. Ducks, pigeons,
pheasants, passerines (perching little birds!) but none are termed birds of prey. Hence, statement 1 is not
correct.
 Generally it is agreed that there are three defining factors that make a bird of prey, it eats meat,
sometimes hunted, sometimes carrion, it has a superb vision with a wide binocular range, and it catches
its food with its feet. Hence, statement 2 is correct.
 There are the diurnal birds of prey – that is birds of prey that fly and hunt during the daylight
hours, hawks, buzzards, eagles and falcons to name but a few examples. There are also the nocturnal
birds of prey (owls) – those that fly and hunt at night and these are the owls. Hence statement 3 is not
correct.
 The victorian scientific name for birds of prey is RAPTOR. This name comes from the latin word
meaning a Plunderer – someone who seizes and carries away. That is pretty much what birds of prey or
‗raptors‘ do – they seize, or grab their prey and carry it away.
 To do this raptors have very strong legs and powerful feet with very sharp curved talons. They also
have strong hooked beaks. Parrots have even stronger hooked beaks but rarely use them for tearing meat,
3 www.visionias.in ©Vision IAS
instead they are used for opening hard fruit and nuts. Although other birds have very strong feet, such as
an Ostrich, none can grip like a raptor, nor generally do they have the very sharp claws.
 There are many birds that, like raptors, eat meat. Small birds, robins, thrushes and kingfishers, and larger
birds such as carrion crows, magpies and ravens all eat meat, but they do not use their feet to catch their
quarry – instead they use their beaks. Only raptors hunt, grasp and kill other creatures by using their
powerful feet.
 Impacts of wind turbines on birds of prey:
o One of these impacts is that wind turbines can kill birds when they collide with the moving blades.
This problem is known worldwide, and some types of bird are more vulnerable to this threat than
others.
 Birds of prey, such as eagles, buzzards and vultures, use the same wind resources that turbines need to
operate. These large soaring birds use the wind to help power their own flight, using updraughts and
thermals to gain height.
 This can make them particularly vulnerable to collisions with wind turbine blades, which can travel at
speeds of up to 290 kilometre / hour and either eagles don‘t see them or don‘t perceive them as a threat
until it is too late.
 In South Africa, recent research found that 36 per cent of birds killed by wind turbines were birds of prey.
These birds have long lifespans and produce relatively few young each year, which means that even a
small increase in deaths can cause their populations to decline.
 This wind-wildlife conflict has been termed a green-green dilemma: More clean energy and healthy bird
populations are both desirable environmental goals, yet with detrimental counter effects.

Q 8.A
 Parliamentary Committees are setup as an instrument to assist the working of Parliament in its various
activities. They are classified into- Standing Committee and Adhoc Committee (temporary). Standing
Committees are permanent, constituted every year and work in a continuous manner. Hence statement 1
is correct.
 It is not mandatory for the bill to be scrutinised by the Parliamentary Standing Committees. In the recent
session of Parliament, all the bills were passed without the scrutiny by parliamentary standing
committees. Hence statement 2 is not correct.
 Moreover, after the formation of the 17th Lok Sabha, parliamentary standing committees have not been
constituted, as consultations among parties are still under way. Partly as a result of this, all the bills were
passed without committee scrutiny.

Q 9.D
 Recent Context- Recently, Kabartal Wetland (Bihar) and Asan Conservation Reserve (Uttarakhand)
have been designated as Ramsar sites, making them ‗Wetlands of International Importance‘. Now total
number of Ramsar sites in India is 39, the highest in South Asia.
 Kabartal Wetland (Bihar)
o Also known as Kanwar Jheel, it covers 2,620 hectares of the Indo-Gangetic plains in the Begusarai
district of Bihar.
o It acts as a vital flood buffer for the region besides providing livelihood opportunities to local
communities.
o Significant biodiversity is present, with 165 plant species and 394 animal species recorded, including
221 bird species. It is also a valuable site for fish biodiversity with over 50 species documented.
o It is an important stopover along the Central Asian Flyway, with 58 migratory waterbirds using it to
rest and refuel.
o Five critically endangered species inhabit the site, including three vultures – the red-headed vulture
(Sarcogyps calvus), white-rumped vulture (Gyps bengalensis) and Indian vulture (Gyps indicus) – and
two waterbirds, the sociable lapwing (Vanellus gregarius) and Baer‘s pochard (Aythya baeri).
o Major threats to the Site include water management activities such as drainage, water abstraction,
damming and canalization.
 Asan Conservation Reserve (Uttarakhand):
o ACR is a 444-hectare stretch of the Asan River running down to its confluence with the Yamuna
River in Dehradun district of Uttarakhand. It is Uttarakhand's first Ramsar Site.
o The damming of the River by the Asan Barrage in 1967 resulted in siltation above the dam wall,
which helped to create some of the Site‘s bird-friendly habitats.
o These habitats support 330 species of birds including the critically endangered red-headed vulture
(Sarcogyps calvus), white-rumped vulture (Gyps bengalensis) and Baer‘s pochard (Aythya baeri).
4 www.visionias.in ©Vision IAS
o Other non-avian species present include 49 fish species, one of these being the endangered Putitora
mahseer (Tor putitora). Fish use the site for feeding, migration and spawning.
 Nandur Madhameshwar bird sanctuary (Maharashtra):
o It is a wetland in Niphad tahsil of Nashik district in Maharashtra. It was declared Ramsar site in
January 2020.
o It has been formed by shallow backwaters of Nandur Madhmeshwar dam and is known as
Maharashtra ‗s Bharatpur.
o It lies in the 100 sq km Nandur Madhameshwar sanctuary formed in 1986.
o It boasts of threatened plants like Indian sandalwood, threatened birds like Indian Spotted Eagle,
Eastern Imperial Eagle, Common Pochard, Bristled Grass Bird, Wooly-necked stork, White-rumped
vulture, Indian vulture and Egyptian vulture, threatened fish Deolali Minnow and mammal like
leopard.
o It houses 536 species of aquatic and terrestrial plants, eight mammal species, 265 bird species, 24
fresh water fish species and 41 butterfly species. It supports migratory birds species by serving as a
wintering and stopover site and as breeding site for resident birds.
 Ramsar Convention on Wetlands is an intergovernmental treaty adopted in 1971. It came into force for
India on 1st February, 1982. Those wetlands which are of international importance are declared as Ramsar
sites.
 The advantages of being declared Ramsar site are that it will help in conservation and wise use of the
wetland, receive national and international cooperation for conservation and management, receive Central
funding, boost tourism, generate employment for locals and bring economic benefits for surrounding areas
due to increased tourism and create awareness for conservation of other wetlands in the state.

Q 10.C
 A Depository Receipt (DR) is a financial instrument representing certain securities (eg. shares, bonds,
etc.) issued by a company/entity in a foreign jurisdiction.
 Securities of a firm are deposited with a domestic custodian in the firm‘s domestic jurisdiction, and
a corresponding ―depository receipt‖ is issued abroad, which can be purchased by foreign
investors. Hence, statement 1 is correct.
 DR is negotiable security (which means an instrument transferrable by mere delivery or by endorsement
and delivery) that can be traded on the stock exchange if so desired. DRs are used for tapping foreign
investors who otherwise may not be able to participate directly in the domestic market.
 For investors, depository receipt is a way of diversifying the risk, by getting exposure to a foreign
market, but without the exchange rate risk as they are foreign currency denominated. Further, they
feel safer to invest in their home location.
 Depending on the location in which these receipts are issued they are called as
o ADRs or American Depository Receipts (if they are issued in the USA on the basis of the
shares/securities of the domestic (say Indian) company),
o IDR or Indian Depository Receipts (if they are issued in India on the basis of the shares/securities of
the foreign company; Standard Chartered issued the first IDR in India).
o in general as GDR or Global Depository Receipt.
 A foreign company can access Indian securities market for raising funds through issue of Indian
Depository Receipts (IDRs). An IDR is an instrument denominated in Indian Rupees in the form of
a depository receipt created by a Domestic Depository (custodian of securities registered with the
Securities and Exchange Board of India) against the underlying equity of issuing company to enable
foreign companies to raise funds from the Indian securities Markets. Hence, statement 2 is correct.
 Also, ADR /GDR issues based on shares of a company are considered as part of Foreign Direct
Investment (FDI) in India, though it is an indirect way of holding shares.

Q 11.A
 Trade winds-
o The trade winds are winds that reliably blow east to west just north and south of the equator.
Hence, statement 1 is correct.
o There is more or less regular inflow of winds from subtropical high pressure belts to equatorial low
pressure belts.
o These tropical winds have north-easterly direction in the northern hemisphere while they are south-
easterly in the southern hemisphere.
o The poleward parts of trade winds or eastern sides of the subtropical anticyclones are
dry because of strong subsidence of air currents from above because of the dominance of
5 www.visionias.in ©Vision IAS
anticyclonic conditions there is strong atmospheric stability, strong inversion of temperature and clear
sky.
o On the other hand, the equatorward parts of the trade winds are humid because they are
characterized by atmospheric instability and much precipitation as the trade while blowing over the
oceans pick up moisture.
o It may be noted that trade winds are more regular and constant over the oceans than over the
lands. Hence, statement 2 is not correct.

Q 12.B
 One of the earliest important rulers of Magadh was the king Bimbisara, who ruled for 52 years from 544
BC to 492 BC. He established the Haryanka dynasty laid the foundations of Magadha by fortification of a
village, which later became the city of Pataliputra. Bimbisara's made Girivraja (Rajgir) as the capital
whereas his son Ajatashatru founded the city of Pataliputra in the 5th century BCE. It was, Udaya
(Udayin), the son of Ajatashatru, who made it the capital of Magadha, which it remained until the
1st century BCE. Hence statement 3 is not correct.
 He pursued a three-pronged policy, namely, matrimonial alliances, friendship with strong rulers
and conquest of weak neighbours to expand the empire. Hence statement 2 is correct.
 Under the policy of matrimonial alliances, he married the sister of Prasenjit, the king of Kosala. She
brought in dowry the territory of Kashi, which yielded a revenue of 1,00,000 coins. The control over Kasi
and friendship with Prasenajit allowed Magadh to concentrate on other areas. His other wives were
daughters of the chiefs of Lichchavi and Madra (middle Punjab) respectively. He also conquered Anga by
defeating its ruler Brahmadatta. Anga and especially its capital Champa were important for the inland and
maritime trade. Thus, Kashi and conquest of Anga became the launching pad for the expansion of
Magadh.
 Bimbisara was contemporary of both Mahavira and Buddha. The Jain texts mention that he was a
disciple of Lord Mahavira. The Buddhist texts mention that he met Buddha before enlightenment. Buddha
promised him to visit his capital after he gets enlightenment. He was a patron of Buddha. Hence
statement 1 is correct.

Q 13.B
 The political events between 1892 to 1905 disappointed the nationalists made them think of more
radical politics called as Extremism or Militant Nationalism.
o The Indian Councils Act of 1892 was a complete disappointment.
o On the other hand, even the existing political rights were attacked.
o In 1897, Natu brothers were deported without being deported, even the charges against them
were not made public.
o In 1898, a law was passed making it an offence to excite feelings of disaffection towards the foreign
government.
o In 1899, the number of Indian members in Calcutta corporation was reduced.
o In 1904, the Official Secrets Act was passed restricting the freedom of the press.
o The anti-congress attitude of Lord Curzon, the then viceroy.
 Even the moderate leader Gopal Krishna Gokhale had commented " Bureaucracy was growing
frankly selfish and openly hostile to nationalist aspirations "
 Neither the socio-cultural aspects were progressive -
o Primary and technical education was not making any advancement
o Indian Universities Act,1904 brought universities under the tight administrative control of
British
o Educated Indians could not find any opportunities leading to rising in unemployment everywhere.
 Hence option (b) is the correct answer.
 Apart from these, growth of self-respect and self-confidence among people, Rise in western
education and hence liberal ideas, International influences like the defeat of Russia by Japan, Italy
by Ethiopia, Boer wars etc., were other factors that contributed to rising of militant nationalism

Q 14.D
 The ingestion of anthropogenic waste, primarily plastic bags and rope by dromedary camels in the United
Arab Emirates, has led to a regional mortality rate of one per cent, according to a recent paper titled, ‗ the
plight of camels eating plastic waste‘ published in the ‗Journal of Arid Environments‘.

6 www.visionias.in
 The research defines ingested waste as a ‗polybezoar‘, a collection of tightly packed indigestible
materials that can include plastics, ropes, other litter and salt deposits trapped in the stomach or
digestive tract forming a large stone-like mass. Hence option (d) is the correct answer.
 Polybeozars lead to gastrointestinal blockages, sepsis from increased gut bacteria, dehydration and
malnutrition in the ruminants (Herbivorous mammals).
 Polybezoars can block intestines and leak toxins, or simply give camels a false sense of fullness, so they
stop eating and slowly starve to death.
 The ingestion of anthropogenic waste, primarily plastic bags and rope by dromedary camels in the United
Arab Emirates, has led to a regional mortality rate of one per cent, according to a recent paper titled, The
plight of camels eating plastic waste published in the Journal of Arid Environments.
 Due to high winds and the open desert environment, the plastic waste escapes open waste bins and
landfills and travels long distances. Improved waste management and alternative systems to package and
deliver goods in the region could help in managing our terrestrial solid waste effectively.

Q 15.D
 An algal bloom or marine bloom or water bloom is a rapid increase in the population of algae in an
aquatic system. Algal blooms may occur in freshwater as well as marine environments.
 Algal blooms are the result of an excess of nutrients (particularly phosphorus and nitrogen) into waters
and higher concentrations of these nutrients in water cause increased growth of algae and green plants.
 Algal blooms are a concern for the ecosystem as some species of algae produce neurotoxins. At the high
cell concentrations reached during some blooms, these toxins may have severe biological impacts on
wildlife. Algal blooms composed of phytoplankters known to naturally produce biotoxins are often called
Harmful Algal Blooms, or HABs.
 It can also be harmful by causing anoxic conditions where oxygen is depleted from the water.
Blooms can block light to organisms lower in the water column, or even clog or harm fish gills.
 Hence, option (d) is the correct answer.

Q 16.A
 Recently, an exploration being carried out by the Geological Survey of India (GSI) has placed the
eastern Himalayan State Arunachal Pradesh on the vanadium map of the country and geologists are
confident of identifying a deposit soon.
 The concentrations of vanadium were found in the palaeo-proterozoic carbonaceous phyllite rocks in
the Depo and Tamang areas of Arunachal Pradesh‘s Papum Pare district. This was the first report of
a primary deposit of vanadium in India with an average grade of 0.76% V2O5 (vanadium pentoxide)
 Vanadium mineralization in Arunachal Pradesh is geologically similar to the ―stone coal‖ vanadium
deposits of China hosted in carbonaceous shale. This high vanadium content is associated with graphite
with a fixed carbon content of up to 16%. The largest deposits are in China, followed by Russia and
South Africa.
 In India, the largest reserves of Vanadium are found in the state of Karnataka followed by
Maharashtra and Odisha. Hence statement 3 is not correct.
 It is recovered as a by-product from the slag collected from the processing of vanadiferous
magnetite ores (iron ore). Hence statement 2 is correct.
 Vanadium in its pure form is a soft, grey, and ductile element primarily derived from mined iron ore,
carbonaceous shale or phyllites, and steel slag.
 It is a high-value metal used in strengthening steel and titanium. Its alloys are durable in extreme
temperatures and environments and are corrosion-resistant. Its addition improves the tensile strength of
steel and of reinforcing bars used for buildings, tunnels, and bridges. Hence statement 1 is correct.
 India is a significant consumer of vanadium but is not a primary producer of the strategic metal.
According to data provided by GSI, India consumed 4% of about 84,000 metric tonnes of vanadium
produced across the globe in 2017. China, which produces 57% of the world‘s vanadium, consumed 44%
of the metal.
Q 17.D
 Recent Context: In the Month of May, 2020, The border tensions between China and India come to the
forefront once again following the ongoing standoff between India and China in the North Sikkim and
Ladakh region across the Line of Actual Control (LAC).
 The stand-off was taken off at Pangong lake in Ladakh on May 5 and at Naku La in Sikkim on May 9
had led to injuries, caused by ―aggressive behaviour on both sides‖. Recent areas of standoff between two
nations are: Aksai Chin, Demchok regions are part of Ladakh (UT) region while Nakula is located in
Sikkim. Hence pairs 2 and 3 are correctly matched.
7 www.visionias.in ©Vision IAS
About India-China border:
The border between India and China is not clearly demarcated throughout and there is no mutually agreed
Line of Actual Control (LAC).The LAC is the demarcation that separates Indian-controlled territory from
Chinese-controlled territory. India considers the LAC to be 3,488 km long, while the Chinese consider it to be
only around 2,000 km. The LAC is divided into three sectors, viz. Western, Middle and Eastern.
 The boundary dispute in the Western Sector (Ladakh) pertains to the Johnson Line proposed by the
British in the 1860s that extended up to the Kunlun Mountains and put Aksai Chin in the then princely
state of Jammu and Kashmir. India used the Johnson Line and claimed Aksai Chin ( Ladakh) as its own.
China, however, do not recognise it and instead accepts McDonald Line which puts Aksai Chin under its
control. Hence pair 3 is correctly matched.
 In the Middle Sector (Himachal Pradesh and Uttarakhand), the dispute is a minor one. Here LAC is
the least controversial except for the precise alignment to be followed in the Barahoti plains. India and
China have exchanged maps on which they broadly agree.
 The disputed boundary in the Eastern Sector (Arunachal Pradesh and Sikkim) is over the McMahon
Line (in Arunachal Pradesh) decided in 1914 in a meeting of Representatives of China, India, and Tibet in
Shimla. Though the Chinese representatives at the meeting initiated the agreement, they subsequently
refused to accept it. The Tawang (Arunachal Pradesh) tract claimed by China was taken over by India
in 1951. Hence pair 1 is correctly matched.

Q 18.B
Payment Banks
 The payments bank will be registered as a public limited company under the Companies Act, 2013, and
licensed under Section 22 of the Banking Regulation Act, 1949, with specific licensing conditions
restricting its activities mainly to acceptance of demand deposits and provision of payments and
remittance services. It will be governed by the provisions of the Banking Regulation Act, 1949; Reserve
Bank of India Act, 1934; Foreign Exchange Management Act, 1999; Payment and Settlement Systems
Act, 2007; Deposit Insurance and Credit Guarantee Corporation Act, 1961.
 The payments bank will be set up as a differentiated bank and shall confine its activities to further
the objectives for which it is set up. Therefore, the payments bank would be permitted to set up its
own outlets such as branches, Automated Teller Machines (ATMs), Business Correspondents (BCs),
etc. to undertake only certain restricted activities permitted to banks under the Banking Regulation Act,
1949.
 Acceptance of demand deposits, i.e., current deposits, and savings bank deposits from individuals, small
businesses, and other entities, as permitted. No NRI deposits should be accepted. The eligible deposits

8 www.visionias.in ©Vision IAS


mobilized by the payments bank would be covered under the deposit insurance scheme of the Deposit
Insurance and Credit Guarantee Corporation of India (DICGC).
 Issuance of ATM/Debit Cards. Payments banks, however, cannot issue credit cards.
 Payments and remittance services through various channels including branches, Automated Teller
Machines (ATMs), Business Correspondents (BCs), and mobile banking are allowed. Hence statement 3
is not correct.
 The payments bank may undertake utility bill payments etc. on behalf of its customers and the general
public.
 The payments bank cannot undertake lending activities or issue loans or credit cards. Hence
statement 2 is correct.
 Apart from amounts maintained as Cash Reserve Ratio (CRR) with RBI on its outside demand and
time liabilities, it will be required to invest minimum 75 percent of its "demand deposit balances" in
Government securities/Treasury Bills with maturity up to one year that is recognized by RBI as
eligible securities for maintenance of Statutory Liquidity Ratio (SLR) and hold maximum 25
percent in current and time/fixed deposits with other scheduled commercial banks for operational
purposes and liquidity management. Hence statement 1 is not correct.

Q 19.C
 Recent Context: The records of the Archaeological Survey of India show that the government has been
able to retrieve 40 art objects between 2014 and 2020. However, demands for the return of objects like
the Kohinoor and the Amravati marbles have been turned down.
 The Amaravati Marbles, also known as Eliot Marbles, are a collection of 120 sculptures and inscriptions
housed in the British Museum in London that were recovered from the Amaravati Mahachaitya in Guntur,
Andhra Pradesh.
 In 1854, Walter Elliot, the commissioner of Guntur (Andhra Pradesh), visited Amaravati and collected
several sculpture panels and took them away to Madras. (These came to be called the Elliot marbles after
him.) He also discovered the remains of the western gateway and came to the conclusion that the structure
at Amaravati was one of the largest and most magnificent Buddhist stupas ever built.
 Hence option (c) is the correct answer.

Q 20.D
 Soil erosion has become now one of the major environmental problems and a serious constraint for
agricultural production. There are many physical and social factors which determine the extent and
severity of soil erosion.
 The principal physical factors are erosivity of rainfall, erodibility of soil, the severity of periodic floods,
length and steepness of the slope.
 The important social factors are deforestation, overgrazing, nature of land use and methods of cultivation.
Ravines, gullies and landslides are the most serious and highly visible forms of land erosion. On the other
hand, sheet erosion caused by rains and erosion due to winds is least visible but equally serious as they too
take a heavy toll of our precious topsoils.
 Soil erosion by ravines and gullies is widespread in India, It has been estimated that 3.67 million hectares
of the soil surface is damaged. There are four major areas of ravines and gullies in India. They are (1)
Yamuna-Chambal ravine zone, (2) Gujarat ravine zone, (3) The Punjab Siwalik foothills zone and (4)
Chhotanagpur zone. Hence options 1, 2 and 3 are correct.
 There are other areas of substantial ravine erosion in the Mahanadi valley, upper Son valley, upper
Narmada and Tapi valleys, Siwalik and Bhabar tract of the western Himalayan foothills and edges of
Ganga Khadar in western Uttar Pradesh.

Q 21.D
 Suhrawardi: This silsilah was founded by Sheikh Shihabuddin Suhrawardi. It was established in
India by Sheikh Bahauddin Zakariya (1182-1262). He set up a leading khanqah in Multan, which was
visited by rulers, high government officials and rich merchants. Sheikh Bahauddin Zakariya openly took
Iltutmisht‘s side in his struggle against Qabacha and received from him the title Shaikhul Islam (Leader of
Islam). It must be noted that unlike the Chishti saints, the Suhrawardis maintained close contacts with
the state. They accepted gifts, jagirs and even government posts in the ecclesiastical department. The
Suhrawardi silsilah was firmly established in Punjab and Sind. Besides these two silsilahs there were
others such as the Firdawsi Silsilah, Shattari Silsilah, Qadiri Silsilah, Naqshbandi Silsilah. Hence, option
(d) is the correct answer.
9 www.visionias.in ©Vision IAS
 Chishti: The Chishti order was founded in a village called Khwaja Chishti (near Herat). In India, the
Chishti silsilah was founded by Khwaja Muinuddin Chishti (born c. 1142) who came to India around
1192. He made Ajmer the main centre for his teaching. He believed that serving mankind was the best
form of devotion and therefore he worked amongst the downtrodden.
 Qalandars, loosely organized group of wandering Muslim dervishes who form an ―irregular‖ (bī-sharʿ) or
antinomian Ṣūfī mystical order. The Qalandarīyah seem to have arisen from the earlier Malāmatīyah in
Central Asia and exhibited Buddhist and perhaps Hindu influences. The adherents of the order were
notorious for their contempt for the norms of Muslim society, their use of drugs, and their coarse
behaviour. They shaved their heads, faces, and eyebrows, dressed only in blankets or in hip-length
hairshirts, led a wandering, nomadic life, and regarded all acts as lawful.
 The Sihilah-i-Khwajgan known as the Naqshbandi Sufi Silsilah in India originated in Transoxiana
(Turkey) under the foundership of Khwajah Abu Yousuf Hamdani (d.c.e. *f14G), Khwajah Bau'd-din
Naqshband (c. e. 1318) was the seventh in the Naqshbandi series. He was adopted by Khwajah
Muhammad Shamsi, the fifth descendent of Hamdani. Akbar's reign brought a flourishing period of the
Naqshbandi order in India. A number of scholars of the order migrated to India and settled here.

Q 22.B
 British poet and diplomat Edward Robert Bulwer-Lytton was the Viceroy of India between 1876 and
1880. He was appointed by Prime Minister Benjamin Disraeli, a Conservative, at a time of intense
competition between Britain and Russia over control of Central Asia. During his viceroyalty, Lytton
worked to improve the Indian administration and supervised his government‘s much-criticized response
to the Great Famine of 1876–78.
 During the 19th century, the problem of Indo-Afghan relations got inextricably mixed up with the Anglo-
Russian rivalry. Britain was expanding colonial power in West, South, and East Asia, Russia was an
expanding power in Central Asia and desired to extend its territorial control in West and East Asia.
 The British wanted to make Afghanistan a subsidiary state whose foreign and defence policies would be
definitely under British control so that it could serve as a base for British expansion in Central Asia.
 Lytton Followed forward policy towards Afghanistan. It is often called the policy of Proud
Reserve. It aimed at having scientific frontiers and safeguarding spheres and influence. He proposed to
effect "the gradual disintegration and weakening of the Afghan power." It led to Second Afghan War
(1878 – 1880) and Treaty of Gandamak was signed. The British secured certain border districts, the
right to keep a Resident at Kabul, and control over Afghanistan's foreign policy.
 However, the British success was short-lived. The national pride of the Afghans had been hurt and once
again they rose to defend their independence. On 3 September 1879, the British Resident, Major
Cavagnari, and his military escort were attacked and killed by rebellious Afghan troops. Afghanistan was
again Invaded and occupied.
 Lord Lytton was severely criticised for his aggressive policy towards Afghanistan and was replaced by
new Viceroy Lord Ripon in 1880.
 Ripon rapidly reversed Lytton's aggressive policy and went back to the policy of non-interference in the
internal affairs of a strong and friendly Afghanistan.
 Hence option (b) is the correct answer.

Q 23.A
 Recent Context: The Reserve Bank of India recently revised priority sector lending guidelines to
include entrepreneurship and renewable resources, in line with emerging national priorities.
 The provisions of PSL apply to every Commercial Bank including Regional Rural Bank (RRB),
Small Finance Bank (SFB), Local Area Bank and Primary (Urban) Co-operative Bank (UCB) other
than Salary Earners' Bank licensed to operate in India by the Reserve Bank of India. Hence statement
1 is correct.
 All scheduled commercial banks and foreign banks (with a sizeable presence in India) are mandated to set
aside 40% of their Adjusted Net Bank Credit (ANDC) for lending to these sectors. Regional rural
banks (RRB), co-operative banks and small finance banks have to allocate 75% of ANDC to PSL. Hence
statement 2 is not correct.
 Priority sector includes the following sectors: Agriculture, Micro, Small and Medium Enterprises
(MSMEs), Export Credit, Education, Housing, Social Infrastructure, Renewable Energy, Start
ups and others.
 Banks having any shortfall in lending to priority sector are required to contribute to the Rural
Infrastructure Development Fund (RIDF) established with NABARD and other funds with
10 www.visionias.in ©Vision IAS
NABARD/NHB/SIDBI/MUDRA Ltd., as decided by Reserve Bank from time to time. Hence statement
3 is correct.
 Besides RBI has devised a mechanism called Priority Sector Lending Certificates (PSLCs) to enable
banks to achieve the priority sector lending target by the purchase of these instruments in the event of a
shortfall. This also incentivizes surplus banks as it allows them to sell their excess achievement over
targets thereby enhancing lending to the categories under priority sector. Under the PSLC mechanism,
the seller sells fulfilment of priority sector obligation and the buyer buys the obligation with no
transfer of risk or loan assets.

Q 24.D
 Recently, the report of the internal working group set up by the RBI has recommended that well-run large
NBFCs with an asset size of ₹50,000 crore and above, including those owned by a corporate house, may
be considered for conversion into banks, subject to completion of 10 years of operations.
 A Non-Banking Financial Company (NBFC) is a company registered under the Companies Act,
1956 engaged in the business of loans and advances, acquisition of shares/stocks/bonds/debentures/
securities issued by Government or local authority or other marketable securities of a like nature, leasing,
hire-purchase, insurance business, chit business but does not include any institution whose principal
business is that of agriculture activity, industrial activity, purchase or sale of any goods (other than
securities) or providing any services and sale/purchase/construction of the immovable property.
 A non-banking institution which is a company and has principal business of receiving deposits under any
scheme or arrangement in one lump sum or in instalments by way of contributions or in any other manner
is also a non-banking financial company.
 RBI does not regulate all NBFCs, some have been exempted from the requirement of registration
under Section 45-IA of the RBI Act, 1934 subject to certain conditions. Hence statement 1 is not
correct.
 Housing Finance Companies are regulated by National Housing Bank, Merchant Banker/Venture Capital
Fund Company/stock-exchanges/stock brokers/sub-brokers are regulated by Securities and Exchange
Board of India, and Insurance companies are regulated by Insurance Regulatory and Development
Authority. Similarly, Chit Fund Companies are regulated by the respective State Governments and Nidhi
Companies are regulated by Ministry of Corporate Affairs, Government of India. Companies that do
financial business but are regulated by other regulators are given specific exemption by the Reserve Bank
from its regulatory requirements for avoiding duality of regulation.
 NBFCs whose asset size is of ₹ 500 cr or more as per last audited balance sheet are considered as
systemically important NBFCs. The rationale for such classification is that the activities of such NBFCs
will have a bearing on the financial stability of the overall economy. Hence statement 2 is not correct.
 As per RBI guidelines, all non-deposit taking NBFCs with asset size of ₹ 10,000 crores and above,
and all deposit-taking NBFCs irrespective of their asset size, shall maintain a liquidity buffer in
terms of LCR which will promote resilience of NBFCs to potential liquidity disruptions by ensuring that
they have sufficient High-Quality Liquid Asset (HQLA) to survive any acute liquidity stress scenario
lasting for 30 days. Hence statement 3 is not correct.

Q 25.A
 The Narmada Valley dry deciduous forests are a tropical dry forest ecoregion of central India. The
eco-region lies mostly in Madhya Pradesh state but extends into portions of Chhattisgarh,
Maharashtra, and Uttar Pradesh states.
 The Narmada Valley dry deciduous forests cover an area of 169,900 sq. km (65,600 sq mi) of the lower
Narmada River Valley and the surrounding uplands of the Vindhya Range to the north and the western
end of the Satpura Range to the south.
 The Narmada Valley is an east-west flat-bottomed valley, or graben, that separates the two plateaus. The
Vindhya Range separates the valley from the Malwa plateau and Bundelkhand upland to the north. The
Satpura Range reaches a height of 1,300m and encloses the valley on the south separating it from the
Deccan plateau.
 The ecoregion includes the western portion of the Satpuras and also extends to the southeast along the
eastern flank of the Western Ghats' range. The uplands of this ecoregion are the northern limits of the
Indian peninsula.
 This area is densely populated and only about 30% of the ecoregion is covered in relatively intact
vegetation, but this does include some large blocks of habitat in the Amarkantak, Vindhya, and Satpura
ranges which are important for the preservation of the tiger.
11 www.visionias.in ©Vision IAS
 Protected areas: About 5% of the ecoregion (7,500 km²) lies within protected areas, the largest of which
are Melghat Tiger Reserve and Nauradehi Wildlife Sanctuary while others include Bandhavgarh,
Panna, and Guru Ghasidas National Park (Sanjay national park).
 Andhari river is a minor river of the Wainganga basin. It flows through the Chandrapur district of
Maharashtra. Meandering through the Tadoba forests, it gives its name to Tadoba Andhari Tiger
Project i.e., Tadoba National Park. Hence the correct answer is option (a).

Q 26.B
 Recent Context: Keeping in mind the need for having a dedicated national fund with the primary
objective of dealing with any kind of emergency or distress situation, like posed by the COVID-19
pandemic and to provide relief to the affected, a public charitable trust under the name of ‗Prime
Minister‘s Citizen Assistance and Relief in Emergency Situations Fund‘ (PM CARES Fund)‘ has been set
up. It is PM CARES registered as a Public Charitable Trust. The trust deed of PM CARES Fund has been
registered under the Registration Act, 1908.
 Who are the Trustees of the PM CARES Fund?
o The Prime Minister is Chairperson (ex-officio) of the PM CARES Fund and Minister of Defence,
Minister of Home Affairs and Minister of Finance, Government of India are ex-officio
Trustees of the Fund.
o PM (Chairman) can also nominate three trustees to the Board who shall be eminent persons in the
field of research, health, science, social work, law, public administration and philanthropy. Hence
statement 1 is not correct.
 The primary objectives of the PM CARES Fund Trust are: -
o To undertake and support relief or assistance of any kind relating to a public health emergency or
any other kind of emergency, calamity or distress, either man-made or natural, including the
creation or upgradation of healthcare or pharmaceutical facilities, other necessary infrastructure,
funding relevant research or any other type of support. Hence statement 2 is correct.
o To render financial assistance, provide grants of payments of money or take such other steps as may
be deemed necessary by the Board of Trustees to assist the affected population.
 Important Features of the fund:
o PM CARES Fund accepts voluntary contributions by individuals/organizations as well as
contributions as part of CSR from Companies/Public Sector Undertakings (PSUs). However, does not
get any budgetary support.
o Any contribution made to the PM CARES Fund by any Company or a PSU shall qualify as
Corporate Social Responsibility (CSR) expenditure as provided under the Companies Act, 2013.
o All contributions towards the PM CARES Fund are 100% exempt from Income Tax under Section
80(G) of the Income Tax Act, 1961 for those who opt for the old tax slabs.
o The PM CARE fund is exempted from paying Income Tax as per Section 10(23)(c) of Income Tax
Act, 1961.
o PM CARES Fund has received exemption from operation of all provisions of the Foreign
Contribution (Regulation) Act, 2010. Foreign Donation is accepted from individuals and
organizations based in Foreign countries through Foreign Credit / Debit Cards and also through Wire
Transfer/SWIFT following the link on Home page of PM CARES Fund portal.
o PM CARES Fund is audited by an independent auditor. Trustees of the Fund during the 2nd
meeting held on 23.04.2020 decided to appoint M/s SARC & Associates, Chartered Accountants,
New Delhi as the auditors of PM CARES Fund for 3 years (Not CAG). This is the same firm, headed
by Sunil Kumar Gupta, which audits the PM National Relief Fund. Hence statement 3 is not
correct.

Q 27.C
 Blood cells are the cells which are produced during hematopoiesis and found mainly in the blood. Blood
is composed of the blood cells which accounts for 45% of the blood tissue by volume, with the remaining
55% of the volume composed of plasma, the liquid portion of the blood.
 There are three types of blood cells. They are:
o Red blood cells (Erythrocytes). Hence, option 4 is not correct.
o White blood cells (Leukocytes)
o Platelets (Thrombocytes)
 What are White Blood Cells?
o White blood cells account for only about 1% of our blood, but their impact is big.

12 www.visionias.in ©Vision IAS


o White blood cells are also called leukocytes. They protect us against illness and disease.
o White blood cells are made in the bone marrow.
o They are stored in our blood and lymph tissues.
o Because some white blood cells have a short life of 1 to 3 days, your bone marrow is always making
them.
 What are the types of WBC?
o Monocytes. They have a longer lifespan than many white blood cells and help to break down
bacteria.
o Lymphocytes. They create antibodies to fight against bacteria, viruses, and other potentially harmful
invaders. Hence, option 2 is correct.
o Neutrophils. They kill and digest bacteria and fungi. They are the most numerous type of white blood
cell and your first line of defense when infection strikes. Hence, option 1 is correct.
o Basophils. These small cells seem to sound an alarm when infectious agents invade your blood. They
secrete chemicals such as histamine, a marker of allergic disease, that help control the body's immune
response. Hence, option 3 is correct.
o Eosinophils. They attack and kill parasites and cancer cells, and help with allergic responses.

Q 28.A
 Legumes are plants that has its seeds in a pod, such as the bean or pea. They are known to increase soil
fertility through their capacity to fix atmospheric nitrogen hence soil fertility can be improved by
inclusion of legume in the cropping system. Hence statement 1 is correct.
 Legumes also help in hydrolyzing organic and inorganic insoluble phosphorus compounds to soluble p
form that can be easily assimilated by plants. Hence statement 2 is correct.
 It does not stable instead increase soil microbial activity. Hence statement 3 is not correct.
 It helps improving soil structure, reduce disease incidence and increase mycorrhizal colonisation which
helps in the uptake of soil mineral nutrients. Hence statement 4 is not correct.

Q 29.D
 The Gandhi-Nehru duo had a tremendous impact on India's civilization and polity.
 Nehru met Gandhi for the first time in 1916. That was the year of his marriage with Kamla Kaul (later
Nehru). In his autobiography, Nehru writes " My first meeting with Gandhiji was about the time of
Lucknow Congress during the Christmas, 1916‘. Nehru's admiration for Gandhi grew when the latter set
up a Satyagraha Sabha in 1919 to defy the notorious Rowlatt Act and two years after in 1921 launched the
non-cooperation movement. Nehru eulogised the Satyagraha movement and non-cooperation movement
of Gandhi. Gandhi and Nehru were completely different people as regards their social status, age, way of
thinking and individuality. Each of these two men had his own world outlook. There were always deep
ideological differences between them.
 Gandhi was a staunch critic of western civilization based on technology. He wanted to preserve his
country from the curse of commercialisation, the horror of machine exploitation and production, the
slavery of the wage labour, the whole black systems of capitalist life. He favoured small scale and cottage
industries including Khadi. His intention was to provide employment to all and thereby solve the problem
of poverty and unemployment. Nehru was enamoured of western science and technology. He
supported heavy and large scale industrialisation. In his autobiography, he wrote, "we cannot stop the
river of change or cut ourselves adrift from it and psychologically we who have eaten the apple of Eden
cannot forget the taste and go back to primitiveness." Hence option 1 is not correct.
 In their attitude on life, Nehru and Gandhi differed from each other. Nehru was absolutely secular
and scientific whereas Gandhi was out and out a man of religion. For Gandhi, religion and morality
constituted the whole of life. They are inseparable. He laid great stress on truth and nonviolence and
expected the Congress to be instrumental for the moral regeneration of the country. Nehru attached much
importance to moral values but not so much to religion. Nehru, religion was a woman's affair. He wanted
Congress to play the role effectively in the political and economic sphere. Hence option 2 is not correct.
 To Nehru, Parliamentary system was the ideal statecraft and democratic practices. Gandhi
disapproved of the British parliament. So far as the general aims and ideals of education for the
improvement of the individual outlook are concerned, there is hardly any difference between Gandhi and
Nehru, but Gandhi believed in democratic decentralisation and grassroots-level governance. Hence
option 3 is not correct.

13 www.visionias.in ©Vision IAS


Q 30.C
 Wholesale Price Index (WPI) amounts to the average change in prices of commodities at the wholesale
level. WPI with 2011-12 base year does not include taxes in order to remove the impact of fiscal
policy. The exclusion of indirect taxes would also ensure the continuity and compatibility of the new
WPI series with Goods and Services Tax (GST). While CPI indicates the average change in the
prices of commodities, at the retail level and include indirect taxes. Inclusion of Indirect tax results in
the divergence between two indices. Therefore, CPI is accepted by RBI to control inflation. Hence
statement 2 is correct.
 The reasons for the divergence between the two indices can also be partly attributed to the difference in
the weight of the food group in the two baskets. CPI Food group has a weight of 39.1 per cent as
compared to the combined weight of 24.4 per cent (Food articles and Manufactured Food products)
in WPI basket. The CPI basket consists of services like housing, education, medical care, recreation
etc. which are not part of WPI basket. A significant proportion of WPI item basket represents
manufacturing inputs and intermediate goods like minerals, basic metals, machinery etc. Hence
statement 1 is correct.

Q 31.D
 A new political thrust was created in the years between 1875 and 1885, leading up to the founding of the
Indian National Congress. This was the creation of the younger, more radical nationalist intellectuals most
of whom entered politics during this period. They established new associations, having found that the
older associations were too narrowly conceived in terms of their programmes and political activity as well
as social bases. For example, the British Indian Association of Bengal had increasingly identified itself
with the interests of the zamindars and, thus, gradually lost its anti-British edge. The Bombay Association
and Madras Native Association had become reactionary and moribund.
 So, the younger nationalists of Bengal, led by Surendranath Banerjea and Anand Mohan Bose, founded
the Indian Association in 1876.
 Younger men of Madras — M. Viraraghavachariar, G. Subramaniya Iyer, P. Ananda Charlu and others —
formed the Madras Mahajan Sabha in 1884.
 In Bombay, the more militant intellectuals like K.T. Telang and Pherozeshah Mehta broke away from
older leaders like Dadabhai Framji and Dinshaw Petit on political grounds and formed the Bombay
Presidency Association in 1885.
 Hence option (d) is the correct answer.

Q 32.A
 The Essential Commodities Act (ECA) was enacted by the Central Government in 1955 to control and
regulate trade and prices of commodities declared essential under the Act.
 The measures that can be taken under the provision of the Act include, among others, licensing,
distribution and imposing stock limits. The governments also have the power to fix price limits, and
selling the particular commodities above the limit will attract penalties. Hence statement 1 is correct and
statement 2 is not correct.
 If the Centre finds that a certain commodity is in short supply and its price is spiking, it can notify stock-
holding limits on it for a specified period.
 The enforcement/implementation of the provisions of the Essential Commodities Act, 1955 lies with the
State Governments and UT Administrations.
 At present, there are seven (7) commodities scheduled under the EC Act, 1955 as essential.
o Drugs;
o Fertilizer, whether inorganic, organic or mixed;
o Foodstuffs, including edible oilseeds and oils;
o Hank yarn made wholly from cotton;
o Petroleum and petroleum products;
o Raw jute and jute textile;
o (i) seeds of food-crops and seeds of fruits and vegetables; (ii) seeds of cattle fodder; (iii) jute seeds;
and (iv) cotton seed.
 With the amendment to the Legal Metrology (Packaged Commodities) Rules 2011 the Government can
fix the retail price of any packaged commodity that falls under the ECA.
 Minimum Export Price (MEP) is the price below which an exporter is not allowed to export the
commodity from India. MEP is imposed in view of the rising domestic retail / wholesale price or
production disruptions in the country. MEP is a kind of quantitative restriction to trade. The government
imposed the Minimum Export Price on Onion under the provisions of The Foreign Trade (Development
14 www.visionias.in ©Vision IAS
And Regulation) Act, 1992. As per section 5 of The Foreign Trade (Development And Regulation) Act,
1992, the Central Government may, from time to time, formulate and announce by notification in the
Official Gazette, the export and import policy and may also, in the like manner, amend that policy. Hence
statement 3 is not correct.

Q 33.D
 The institution of the Iqta had been in force in the early Islamic world as a form of reward for services to
the state. In the caliphate administration, it was used to pay civil and military officers. After the
establishment of the Sultanate in India, iqta system was introduced by the Sultans. To begin with,
the army commanders and nobles were given territories to administer and collect the revenue. The
territories thus assigned were called iqta and their holders as iqtadar or muqti. Hence statement 1 is not
correct.
 In essence, this was a system of payment to the officers and maintenance of army by them. From the 14th
century, we hear of Walis or muqtis who are commanders of military and administrative tracts called Iqta.
The duty of the muqtis was to lead military campaigns and maintain law and order in their iqtas. In
exchange for their military services, the muqtis collected the revenues of their assignments as salary. They
also paid their soldiers from these revenues.
 The iqtas were of varying sizes and were given also to the nobles for purposes other than military and
administrative. Iqtas were granted for the maintenance of religious houses, tombs of saints and Sultans,
mosques and also to persons for their livelihood who were devoted to religious or literary pursuits. Hence
statement 2 is not correct.
 Control over muqtis was most effective if their office was not inheritable and if they were assigned iqtas
for a short period of time before being shifted. These harsh conditions of service were rigorously imposed
during the reigns of Alauddin Khalji and Muhammad Tughluq. Accountants were appointed by the state
to check the amount of revenue collected by the muqtis. Care was taken that the muqti collected only the
taxes prescribed by the state and that he kept the required number of soldiers.
 During the reign of Muhmmad-bin-Thughlaq, a number of governors were appointed on revenue sharing
terms where they were to give a fixed sum to the state. During the time of Feroze Shah Tughlaq, the
control of the state over iqta was diluted when iqtas became hereditary.

Q 34.A
 Recently the Haryana Forest Department has started aerial seeding across the state on a pilot
basis. This technique will allow plantation in sections of the Aravallis that are either difficult to access or
inaccessible altogether, and the pilot project will help determine the effectiveness of the technology and
the dispersal mechanism.
 Statement 1 is correct: Aerial seeding is a technique of plantation in which seed balls – seeds covered
with a mixture of clay, compost and other components – are sprayed on the ground using aerial devices,
including planes, helicopters or drones. Areas that are inaccessible, have steep slopes, are fragmented
or disconnected with no forest routes, making conventional plantation difficult, can be targeted with
aerial seeding. Furthermore, the process of the seed‘s germination and growth is such that it
requires no attention after it is dispersed – the reason why seed pellets are known as the ―fire and
forget‖ way of plantation.
 Aerial seeding is suitable for the following conditions;
o Remote and inaccessible areas.
o Ragged areas.
o Region where labour scarcity or sparse population is available.
o Deep ravines where afforestation is not possible through manual mode.
o Area denuded by clear-cutting or shifting cultivation.
o Areas devastated by pest and diseases, volcanoes, earthquakes and landslides. Hence, statement 2 is
not correct.
Q 35.C
 Two opposing air masses must have contrasting temperatures e.g. one air mass should be cold, dry and
dense while the other should be warm, moist and light. In such conditions, when two air masses converge,
then cold and denser air mass invades the area of warm and light air mass and pushes it upward and thus
front is formed.
 Inspite of convergence of two air masses at the equator no front is formed due to uniform temperature
conditions of two air masses (trade winds). Hence, statement 1 is correct.
 Convergent circulation involves meeting of winds at common point from all directions while divergent
circulation refers to spread of winds outward in all directions from a central point.
15 www.visionias.in ©Vision IAS
 Though temperature difference is produced in convergent circulation but this situation occurs at a point
and not along a line thus front cannot be created because for frontogenesis it is necessary that
temperature difference does exist along a line and not at point.
 In divergent circulation front is formed because temperature difference does exist along a line. Hence,
statement 2 is not correct.
 A cold front is associated with bad weather characterized by thick clouds, heavy downpour with
thunderstorms, lightning etc. Sometimes cold frontal precipitation is also associated with snowfall and
hailstorms. Hence, statement 3 is correct.

Q 36.B
 Recent Context: The Delhi High Court has refused to scrap the Death Audit Committee (DAC)
constituted by the Delhi government, observing that there seems to be ―no arbitrariness or discrimination
being done nor is there are any material to show false fabrication of data‖ on the number of coronavirus
cases and the death toll.
o A PIL was filed on May 21, 2020 alleging that the Delhi government was downplaying the spread of
the disease and not presenting the true data on the number of people who had died due to Covid. DAC
audits each death of a coronavirus patient in government and private hospitals.
 The constitution of Death/Mortality Audit Committees is a public health strategy undertaken by the health
departments in states to review the circumstances of the death, the cause of death, identify the possible
reasons that led to the death and the factors that could have prevented the death.
 Such committees have both external members and health department officials and are usually
constituted for specific health conditions. Its members are selected by the state government. Hence
statement 2 is correct.
 These committees primarily help in devising strategies for case detection, clinical management, health
system response and their implementation. The committee also reviews the causes of death due to
COVID-19 and ensures that the comorbid conditions are recorded accurately.
 Death/Mortality Audit Committee is a committee set up by respective state governments which
collects data related to deaths in the respective states. These are constituted by the state
governments for auditing and publishing the death data.
 Deaths due to COVID-19 are being recorded as per the database set up by Indian Council of the Medical
Research (ICMR). Hence statement 1 is not correct.

Q 37.A
 The Muddiman Committee, 1924 was a committee appointed by the Government of India in early
1924 with the terms of reference of making an empirical investigation into the working of the
Constitution as set up in 1921 under the India Act of 1919, and making appropriate
recommendations for the consideration of the authorities. The official designation of the committee
was the Reforms Enquiry Committee, but it came to be known as the Muddiman Committee after the
name of its chairman, Sir Alexander Muddiman, who was then a Home Member of the Government of
India.
 The Committee was appointed in 1924 to consider what adjustments were desirable inside the framework
of the Act of 1919, presented a majority and a minority report. The majority consisted of Sir
Alexander Muddiman, Sir Muhammed Shafi, the Maharaja of Burdwan, Sir Arthur Froom and Sir Henry
Moncrieff Smith. Reviewing the evidence, it declared that, while it was too soon to pronounce that
dyarchy had succeeded, it was also premature to say that it had failed, and recommended a few minor
changes, including some small additions to the list of transferred subjects. The minority was composed of
Sir Tej Bahadur Sapru, Sir Sivaswamy Aiyer, Mr Jinnah, and Dr Paranjpye. These gentlemen reported in
effect that no tinkering would mend a situation which called for more radical treatment.
 The majority suggested only minor changes in the structure of the Constitution while the minority,
consisting entirely of non-official Indians, condemned the dyarchy and advocated for its immediate
abolition and democratisation of the Constitution. No effect was however given to the recommendations
of the Committee.
 Hence option (a) is the correct answer.

Q 38.D
 Taxes subsumed in GST:
GST replaces the following taxes previously levied and collected by the Centre:
o Central Excise duty
o Excise Duty levied under the Medicinal and Toilet Preparations (Excise Duties) Act 1955,
16 www.visionias.in ©Vision IAS
o Additional Excise Duties (Goods of Special Importance)
o Additional Excise Duties (Textiles and Textile Products)
o Additional Customs Duty (commonly known as Countervailing duties or CVD)
o Special Additional Duty of Customs (SAD)
o Service Tax
o Cesses and surcharges in so far as they relate to the supply of goods and services
o Taxes on the sale or purchase of newspapers and on advertisements published therein.
 State taxes that would be subsumed within the GST are:
o State VAT/ Sales Tax
o Central Sales Tax (levied by the Center and collected by the States
o Luxury Tax
o Octroi
o Entry Tax i.e., taxes on the entry of goods into a local area for consumption, use or sale therein. (other
than those in lieu of octroi)
o Purchase Tax
o Entertainment Tax which are not levied by the local bodies; i.e. panchayats, municipalities and
District councils of autonomous districts can impose taxes on entertainment and amusements
o Taxes on general advertisements
o Taxes on lotteries, betting and gambling
o State cesses and surcharges insofar as they relate to supply of goods or services
 GST does not subsume stamp duties and customs duties.
 Hence, option(d) is the correct answer.

Q 39.A
 Recent Context: Recently, on 4th May 2020, Indian Prime Minister Narendra Modi participated in the
online Summit of Non-Aligned Movement (NAM) Contact Group to discuss response to the ongoing
COVID-19 pandemic crisis. The online NAM Contact Group Summit on the theme of "United against
COVID-19‖. The summit was hosted by the current Chairman of NAM, President of Republic of
Azerbaijan. Therefore, Azerbaijan holds the NAM presidency for 3 years until the 19th summit in
2022. Hence statement 3 is not correct.
 About Non-Aligned Movement:
o The Non-Aligned Movement was formed during the Cold War as an organization of States that did
not formally align themselves with either the United States or the Soviet Union, but sought to remain
independent or neutral.
o Origin: Bandung Asian-African Conference held in Bandung, Indonesia in 1955, is the most
immediate antecedent to the creation of the Non-Aligned Movement. During the conference "Ten
Principles of Bandung" were proclaimed which were adopted later as the main goals and objectives of
the policy of non-alignment. Hence statement 1 is correct.
 The first conference of NAM which is also known as the Belgrade Conference held in 1961 under the
leadership of Jawaharlal Nehru of India, Josip Broz Tito of Yugoslavia, Gamal Abdel Nasser of Egypt,
Kwame Nkrumah of Ghana, and Sukarno of Indonesia. Though Pakistan attended the Belgrade
conference but didn‘t join the NAM.
 Pakistan started making efforts to join the non-aligned movement in 1969. Since it was then a member of
the CENTO, it was rightly refused entry. It was only after the dissolution of the SEATO in 1972 and the
CENTO in 1979 that the decks were cleared for Pakistan to join the non-aligned movement. Pakistan
participated in the non-aligned conference in Havana in September 1979. Hence statement 2 is not
correct.

Q 40.A
 In India, Spot Exchanges refer to electronic trading platforms that facilitate the purchase and sale of
specified commodities, including agricultural commodities, metals, and bullion by providing spot delivery
contracts in these commodities.
 Advantages of Spot Exchanges:
o Efficient price determination- as the price is determined by a wider cross-section of people from
across the country, unlike the present scenario where price discovery for commodities happens only
through local participation. This also ensures transparency in price discovery.
o Anonymity ensures convergence of different price perceptions, as the buyer or seller merely
expresses their desire to trade without even meeting directly.

17 www.visionias.in ©Vision IAS


o If spot exchanges can ensure participation in large numbers by farmers, traders, and processors across
the country it can eliminate the possibility of cartelization and other such unhealthy practices
prevalent in commodity markets.
o Spot Exchanges can also usher in some best Practices in commodity trading like a system of grading
for quality, creating a network of warehouses with assaying facilities, facilitating trading in relatively
smaller quantities, lower transaction costs, etc.
o Bank finance available against the goods in the warehouse on easier terms improves holding capacity
and can actually incentivize farm production and hence reduce rural poverty.
o Since the trades are guaranteed, counterparty risk is avoided.
o Apart from The National Spot Exchange Ltd, Three players were granted permission: FTIL, NCDEX,
and NMCE; FTIL and NCDEX provided electronic spot trading services.
 Hence option (a) is the correct answer.

Q 41.B
 Recently it was reported that a brood of periodical cicadas, noisy insects that breed underground for as
long as 13-17 years are expected to emerge into some states on the east coast of the US this year.
 Cicadas are insects that spend most of their lives underground and emerge from the soil mainly to
mate. Once out of the ground, their life span is fairly short, somewhere between two-four weeks. At
present, there are about 15 active broods of these cicadas as some have gone extinct. The insects are
found in the America‘s as well as New Zealand and Australia. Hence option (b) is the correct
answer.
 It is not clear why their development period is so long, researchers suspect that it may be linked to
avoiding predators above the soil. During this time underground the nymphs feed on sap from plant roots.
After this developmental period, the cicada nymphs construct a ―cicada hut‖ and burrow their way out
from the soil and climb onto any nearby tree or vegetation.
 What happens when cicadas emerge? After emerging from the ground in billions, the cicadas shed their
exoskeletons or outer skins to take their winged form. Their exoskeletons are frequently found attached to
tree trunks and twigs. The emergence of cicadas is often ―tightly‖ synchronised, with most adults
appearing within a few nights. The lifespan of adult cicadas is short, about two to four weeks during
which time they feed relatively little and mate. Male cicadas ―sing‖ to attract the females, the collective
chorus of these male cicadas is very loud and can reach up to 100 decibels, which is as much as a powered
lawnmower.
 Protozoa (also protozoan, plural protozoans) is an informal term for a group of single-celled eukaryotes,
either free-living or parasitic, which feed on organic matter such as other microorganisms or organic
tissues and debris.

Q 42.B
 Recent Context: During the Covid-19 pandemic, a number of cyberattacks are observed which were in
the form of Ransomware , Phishing , Botnet and Man-in-the-middle attack etc. Phishing: It denotes to the
cyber term of luring and cheating an internet user through a fake SMS or email and thereby breaching
their privacy to steal sensitive information. Hence option (a) is not correct.
 Ransomware: It refers to the malicious software that locks up the files on your computer, encrypts them,
and demands that you pay to get your files back. Wanna Decryptor, or WannaCry, Petya are form of
ransomware. Hence option (b) is the correct answer.
 Botnet Attack: It is a collection of internet-connected devices infected by malware that allow hackers to
control them. Cyber criminals use botnets to instigate botnet attacks, which include malicious activities
such as credentials leaks, unauthorized access, data theft and massive Distributed Denial of Service
(DDoS) attacks. Hence option (c) is not correct. Distributed denial-of-service attacks target websites
and online services. The aim is to overwhelm them with more traffic than the server or network can
accommodate.
 Man-in-the-middle attack : It also known as eavesdropping attacks, occur when attackers insert
themselves into a two-party transaction. Once the attackers interrupt the traffic, they can filter and steal
data. Two common points of entry for MitM attacks:
o On unsecure public Wi-Fi, attackers can insert themselves between a visitor‘s device and the network.
Without knowing, the visitor passes all information through the attacker.
o Once malware has breached a device, an attacker can install software to process all of the victim‘s
information. Hence option (d) is not correct.

18 www.visionias.in ©Vision IAS


Q 43.C
 The Constitution makes it clear that if any question arises whether a matter falls within the governor‘s
discretion or not, the decision of the governor is final and the validity of anything done by him cannot be
called in question on the ground that he ought or ought not to have acted in his discretion. The governor
has constitutional discretion in the following cases:
o Reservation of a bill for the consideration of the President.
o Recommendation for the imposition of the President‘s Rule in the state.
o While exercising his functions as the administrator of an adjoining Union Territory (in case of an
additional charge).
o Seeking information from the chief minister with regard to the administrative and legislative matters
of the state.
 In addition to the above constitutional discretion (i.e., the express discretion mentioned in the
Constitution), the governor, like the president, also has situational discretion (i.e., the hidden discretion
derived from the exigencies of a prevailing political situation) in the following cases:
o Appointment of the chief minister when no party has a clear-cut majority in the state legislative
assembly or when the chief minister in office dies suddenly and there is no obvious successor.
o Dismissal of the council of ministers when it cannot prove the confidence of the state legislative
assembly.
o Dissolution of the state legislative assembly if the council of ministers has lost its majority.
 Article 161 confers ―power‖ on the Governor to grant pardons. It clearly lays down that the Governor has
the power to grant pardons, reprieves, etc. to persons convicted of offences against any law relating to a
matter to which the executive power of the State extends. Article 161 is broadly worded and does not
contain any restriction as to the time at which the said power is exercised or the circumstances or manner
in which it is to be exercised. However, a reference to the interpretation of this provision by the
Supreme Court makes it clear that the power contained therein is to be exercised by the Governor
as per the aid and advice of the council of ministers.
 Hence option (c) is the correct answer.

Q 44.D
 What is styrene? It is a flammable liquid that is used in the manufacturing of polystyrene plastics,
fiberglass, rubber, and latex. Styrene is also found in vehicle exhaust, cigarette smoke, and in natural
foods like fruits and vegetables.
 What happens when exposed to styrene? Short-term exposure to the substance can result in respiratory
problems, irritation in the eyes, irritation in the mucous membrane, and gastrointestinal issues.
And long-term exposure could drastically affect the central nervous system and lead to other related
problems like peripheral neuropathy. It could also lead to cancer and depression in some
cases. Symptoms include headache, hearing loss, fatigue, weakness, difficulty in concentrating etc.
 What is styrene used for? Styrene is the main raw material for synthesis of polystyrene, or (C8H8)n.
Polystyrene, in turn, is a versatile plastic that is used to make parts of various appliances such as
refrigerators or micro-ovens; automotive parts; and parts of electronics such as computers; and also to
manufacture disposable cups and in food packaging. Styrene is also used as an intermediate to produce
copolymers — which are polymers derived from one or more species of monomers such as
styrene. According to Tox Town, a website run by the US National Library of Medicine, styrene is
also found in vehicle exhaust, cigarette smoke, and in natural foods like fruits and vegetables. Hence
option (b) is the correct answer.

Q 45.C
 A study of a microscopic ancient marine algae (Coccolithophores) led by the National Centre for
Polar and Ocean Research (NCPOR) has found that there is a decrease in the concentration of oceanic
calcium carbonate (CaCO3) in the southern Indian ocean.
 This decrease in CaCO3 is attributed to the increase in the concentration of other single-celled algae
known as diatoms. This, in turn, will affect the growth and skeleton structure of coccolithophores, with
potential significance for the world ocean ecosystem.
 Coccolithophores are single-celled algae living in the upper layers of the world‘s oceans. They have
been playing a key role in marine ecosystems and the global carbon cycle for millions of
years. Coccolithophores calcify marine phytoplankton that produces up to 40 per cent of open ocean
calcium carbonate and responsible for 20 per cent of the global net marine primary productivity.
Hence, statement 1 is correct.

19 www.visionias.in ©Vision IAS


 Coccolithophores build exoskeletons from individual CaCO3 plates consisting of chalk and seashells
building the tiny plates on their exterior. Though carbon dioxide is produced during the formation of these
plates, coccolithophores help in removing it from the atmosphere and ocean by consuming it during
photosynthesis. At equilibrium, coccolithophores absorb more carbon dioxide than they produce,
which is beneficial for the ocean ecosystem. Hence, statement 2 is correct.
 NOTE: National Centre for Polar and Ocean Research (NCPOR) is India‘s premier R&D institution
responsible for the country‘s research activities in the Polar and Southern Ocean realms.
 The mandate of NCPOR is multi-dimensional:
 Leadership role in niche areas of scientific research in the domain of polar and ocean sciences.
 Lead role in the geoscientific surveys of the country‘s EEZ and its extended continental shelf
beyond 200M, deep-sea drilling in the Arabian Sea basin through the IODP, exploration for ocean
non-living resources such as the gas hydrates and multi-metal sulphides in mid-ocean ridges.
 Facilitatory role in the scientific research activities being undertaken by several national institutions
and organizations in Antarctica, the Arctic and in the Indian Ocean sector of the Southern
Ocean.
 Management and upkeep of the Indian Antarctic Research Bases ―Maitri‖ and ―Bharati‖, and
the Indian Arctic base ―Himadri‖

Q 46.B
 The British, after imposing the Government of India Act of 1935, decided to immediately put into
practice provincial autonomy, and announced the holding of elections to provincial legislatures in early
1937. After intense debate, the Congress decided at Lucknow in early 1936 and at Faizpur in late
1936 to fight the elections. Thus, provincial elections were held in British India in the winter of 1936-37
as mandated by the Government of India Act 1935. Elections were held in eleven provinces - Madras,
Central Provinces, Bihar, Orissa, United Provinces, Bombay Presidency, Assam, NWFP, Bengal, Punjab
and Sindh. Hence statement 1 is correct.
 The Congress won a massive mandate at the polls despite the narrow franchise. It won 716 out of 1,161
seats it contested. It had a majority in most of the provinces. The exceptions were Bengal, Assam, the
North-Western Frontier Province, Punjab and Sind; and in the first three, it was the largest single
party. Hence statement 2 is not correct.
 The Congress formed Ministries in six provinces: Madras, Bombay, Central Provinces, Orissa, Bihar
and U.P. Later, in the North-West Frontier Province and Assam. To guide and coordinate their
activities, a central control board known as the Parliamentary Sub-Committee was formed, with Sardar
Patel, Maulana Abul Kalam Azad and Rajendra Prasad as members. In the 28 months of Congress rule in
the provinces, reforms in social welfare, agriculture, administration, civil liberties etc. took place.
 There was increased militancy and industrial unrest which clashed with the Congress Governments. The
Left was highly critical of the Congress Governments‘ handling of popular protest. Gandhiji too thought
that the policy of ministry formation was leading to a crisis. Gandhiji repeatedly lashed out in the columns
of Harijan against the growing misuse of office and creeping corruption in Congress ranks. The
Congress ministries resigned in October and November 1939, in protest against Viceroy Lord
Linlithgow's action of declaring India to be a belligerent in the Second World War without
consulting the Indian people. Gandhiji welcomed the resignations for another reason as they would
help cleanse the Congress of the ‗rampant corruption.‘ Hence statement 3 is not correct.

Q 47.A
 Recently, the fifth Scorpene submarine of Project-75 named ‗Vagir‘ has been launched at Mazagon
Dock Shipbuilders Limited (MDL) in Mumbai.
 The Scorpène-class (Kalvari-Class) submarines are a class of diesel-electric attack
submarines jointly developed by Indian and France. Hence statement 1 is correct.
 The submarines are designed by French naval defence and energy company DCNS and are being
manufactured by Mazagon Dock Limited in Mumbai with technology transfer from France. Hence
statement 2 is not correct.
 The other vessels in the class are INS Kalvari, INS Khanderi, INS Karanj, INS Vela and INS Vagsheer.
Of these Kalvari and Khanderi have been commissioned in 2017 and 2019, Vela and Karanj and
undergoing sea trials, Vagir has now been launched and Vagsheer is under construction.
 The submarines in the current Kalvari-class take their names from erstwhile decommissioned classes of
submarines named Kalvari which included Kalvari, Khanderi, Karanj and Vela class — which included

20 www.visionias.in ©Vision IAS


Vela, Vagir, Vagshir. The now-decommissioned Kalvari and Vela classes were one of the earliest of the
submarines in post-independence Indian Navy, which belonged to Soviet origin Foxtrot class of vessels.
 Like Kalvari – which means Tiger Shark, Vagir has been named after a Sand Fish, a predatory
marine species. Khanderi has been named after an Island Fort built by Chhatrapati Shivaji (South
of Mumbai along the coast of Maharashtra), which played a key role in his Navy. Karanj has also
been named after an Island located South of Mumbai. Hence statement 3 is not correct.
 Project 75: The decision to build the six new submarines is part of the 30-year submarine building
programme cleared in 1999. The plan is to have 24 submarines in 30 years.
 Project 75I: Under this project, the modern variants of the Scorpene class of submarines have what is
called the Air Independent Propulsion (AIP) which enables non-nuclear submarines to operate for a long
time without access to surface oxygen. It also needs to be noted that the Defence Research and
Development Organisation (DRDO) has an ongoing programme to build a fuel cell-based AIP system for
Indian Naval Submarines.

Q 48.D
 Financial Market Infrastructure (FMI) refers to critically important institutions responsible for providing
clearing, settlement and recording of monetary and other financial transactions.
 The different categories of FMIs, as identified under Principles for financial market infrastructures
[PFMIs], are-
o Payment Systems (PSS): A payment system is a set of instruments, procedures, and rules for the
transfer of funds between or among participants. The system includes the participants and the entity
operating the arrangement. Payment systems are typically based on an agreement between or among
participants and the operator of the arrangement, and the transfer of funds is effected using an agreed-
upon operational infrastructure.
o Central Securities Depositories (CSD): It provides securities accounts, central safekeeping services,
and asset services, which may include the administration of corporate actions and redemptions. A
CSD can hold securities either in physical form (but immobilised) or in dematerialised form.
o Securities Settlement Systems (SSS): It enables securities to be transferred and settled by book entry
according to a set of predetermined multilateral rules. Such systems allow transfers of securities either
free of payment or against payment. An SSS may be organised to provide additional securities
clearing and settlement functions, such as the confirmation of trade and settlement instructions.
o Central Counterparties (CCP): It interposes itself between counterparties to contracts traded in one
or more financial markets, becoming the buyer to every seller and the seller to every buyer and
thereby ensuring the performance of open contracts. CCPs have the potential to significantly reduce
risks to participants through the multilateral netting of trades and by imposing more effective risk
controls on all participants.
o Trade Repositories (TR): It is an entity that maintains a centralised electronic record (database) of
transaction data. It operates with effective risk controls and serve an important role in enhancing the
transparency of transaction information to relevant authorities and the public, promoting financial
stability, and supporting the detection and prevention of market abuse.
 Though the expression ―Financial Market Infrastructure‖ is not specifically defined in the Payment and
Settlement Systems (PSS) Act, the definition of payment systems in the PSS Act includes all categories of
FMIs as indicated in the PFMI report except for the Trade Repository.
 The Reserve Bank maintains a separate list of authorized payment systems which are declared as FMIs
and are made public.
 It includes Real Time Gross Settlement System (RTGS), Securities Settlement Systems (SSS),
Clearing Corporation of India Ltd (CCIL) and Negotiated Dealing System- Order Matching (NDS-
OM).
 SEBI list the following as FMIs:
o Indian Clearing Corporation Ltd. (ICCL)
o MCX-SX Clearing Corporation Ltd. (MCX-SXCCL)
o National Securities Clearing Corporation Ltd. (NSCCL)
o Central Depository Services Ltd. (CDSL)
o National Securities Depository Ltd (NSDL)
o National Commodity & Derivatives Exchange Limited (NCDEX)
o Multi Commodity Exchange of India Limited (MCX)

21 www.visionias.in ©Vision IAS


Q 49.A
 The Civil Disobedience Movement was started by Gandhiji on March 12, 1930, with his famous
Dandi March. Gandhiji walked nearly 200 miles from Sabarmati Ashram to Dandi together with 78 of
his chosen followers where Gandhiji and his followers made salt in violation of the salt laws. It was
symbolic of the Indian people's refusal to live under British-made laws. The movement spread rapidly to
different parts of the country. People joined strikes, demonstrations, and the campaign to boycott foreign
goods and to refuse to pay taxes.
 Various Movements in the Civil Disobedience Movement:
o Defiance of Forest Laws assumed a mass character in Maharashtra, Karnataka and the Central
Provinces, especially in areas with large tribal populations who had been the most seriously affected
by the colonial Government‘s restrictions on the use of the forest. At some places, the size of the
crowd that broke the forest laws swelled to 70,000 and more.
o Cunningham Circular played a very important role in shaping the freedom struggle in Assam.
While the entire country was getting ready for the start of the Civil Disobedience Movement in
1930, students in Assam played a key role in the freedom struggle. To stem the students'
participation in the Civil Disobedience Movement, the 'Cunning Circular' was implemented in
1930 by the British. This ruling forbade students from participating in political activities. A powerful
agitation led by students was launched against the infamous ‗Cunningham Circular‘ which forced
students and their guardians to furnish assurances of good behaviour. U.P. was the setting of another
kind of movement: a no revenue, no-rent campaign. The no-revenue part was a call to the zamindars
to refuse to pay revenue to the Government, the no-rent a call to the tenants not to pay rent to the
zamindars. In effect, since the zamindars were largely loyal to the Government, this became a no-rent
struggle. Hence option 2 is correct.
o Eastern India became the scene of a new kind of no-tax campaign: refusal to pay the chowkidar
tax. Chowkidars, paid out of the tax levied especially on the villages, were guards who supplemented
the small police force in the rural areas in this region. They were particularly hated because they acted
as spies for the Government and often also as retainers for the local landlords. Hence option 1 is
correct.
 Individual Satyagraha
o In 1940, then Viceroy Linlithgow made a set of proposals called the ‗August offer‘ which
included that a representative Indian body would frame a constitution for India after the war and
Dominion status is the objective for India.
o Indian National Congress rejected this offer in its meeting at Wardha in August 1940 and
demanded complete freedom from the colonial rule.
o Jawaharlal Nehru remarked that the dominion status concept was as dead as a doornail. After
this, Mahatma Gandhi initiated the Individual Satyagraha to affirm the right to free speech. He
avoided a mass satyagraha because he did not want violence. Thus, Individual Satyagraha was
launched long after Civil Disobedience came to an end. Hence option 3 is not correct.

Q 50.B
 What Is Photosynthesis?
o Photosynthesis is the process used by green plants and a few organisms that use sunlight, carbon
dioxide and water to prepare their food.
o The process of photosynthesis is used by plants, algae and certain bacteria that convert light energy
into chemical energy. The glucose formed during the process of photosynthesis provides two
important resources to organisms: energy and fixed carbon.
 Site of Photosynthesis:
o Photosynthesis takes place in special organelles known as chloroplast. This organelle has its own
DNA, genes and hence can synthesize its own proteins. Chloroplasts consist of stroma, fluid, and a
stack of thylakoids known as grana. There are three important pigments present in the chloroplast that
absorb light energy, chlorophyll a, chlorophyll b, and carotenoids.
 The following events occur during this process:
o Absorption of light energy by chlorophyll.
o Conversion of light energy to chemical energy and splitting of water molecules into hydrogen and
oxygen. Hence, statement 1 is not correct and 2 is correct.
o Reduction of carbon dioxide to carbohydrates. Hence, statement 3 is not correct.

22 www.visionias.in ©Vision IAS


Q 51.B
 A Notified Area Committee is created for the administration of two types of areas–a fast developing
town due to industrialisation, and a town which does not yet fulfil all the conditions necessary for
the constitution of a municipality, but which otherwise is considered important by the state
government.
 Since it is established by notification in the government gazette, it is called a notified area committee.
Though it functions within the framework of the State Municipal Act, only those provisions of the act
apply to it which are notified in the government gazette by which it is created.
 It may also be entrusted to exercise powers under any other act. Its powers are almost equivalent to those
of a municipality. But unlike the municipality, it is an entirely nominated body, that is, all the members of
a notified area committee including the chairman are nominated by the state government. Thus, it is
neither an elected body nor a statutory body.
 Hence, option (b) is the correct answer.

Q 52.D
 Article 123 of the Constitution empowers the President to promulgate ordinances during the recess of
Parliament. Ordinance-making power is the most important legislative power of the President. It has been
vested in him to deal with unforeseen or urgent matters. But, the exercises of this power is subject to the
following limitations:
o He can promulgate an ordinance only when either of the two Houses of Parliament is not in session or
when both the Houses of Parliament are not in session. An ordinance can also be issued when only
one House is in session because a law can be passed by both the Houses and not by one House
alone. An ordinance made when both the Houses are in session is void. Hence, statement 1 is
correct.
o He can make an ordinance only when he is satisfied that the circumstances exist that render it
necessary for him to take immediate action. In Cooper case, (1970), the Supreme Court held that
the President‘s satisfaction can be questioned in a court on the ground of malafide. Thus, the
President‘s satisfaction is justiciable on the ground of malafide. Hence, statement 2 is not
correct.
o His ordinance-making power is coextensive as regards all matters except duration, with the law-
making powers of the Parliament. This has two implications:
 An ordinance can be issued only on those subjects on which the Parliament can make laws.
 An ordinance is subject to the same constitutional limitation as an act of Parliament. Hence, an
ordinance cannot abridge or take away any of the fundamental rights.
o Every ordinance issued by the President during the recess of Parliament must be approved by both the
Houses. If Parliament takes no action at all, the ordinance ceases to operate on the expiry of six weeks
from the reassembly of Parliament.
o An ordinance like any other legislation may modify or repeal any act of Parliament or another
ordinance. It can alter or amend a tax law also. However, it cannot be issued to amend the
Constitution. Hence, statement 3 is correct.

Q 53.D
 The ecological importance of insects cannot be underestimated: They form the basal part of the food
pyramid and impact our agriculture ecosystems as well as human health.
o Their extinction can have a cascading effect on the upper levels of the food pyramid. Insects are
increasingly susceptible to extinction due to increasing climate crisis. Rampant and indiscriminate use

23 www.visionias.in ©Vision IAS


of chemicals in commercial agricultural practises, mainly monocropping systems, has been taking a
toll on insects in the vicinity of farmlands and plantations.
 There are five major ecosystem services provided by insects:
o Decomposers. By recycling of dung, dead wood material (e.g. fallen tree trunks or foliage) or
carcasses of dead animals (e.g., burying beetles, see on the Figure), insects stimulate and accelerate
breakdown of organic materials by such organisms as soil mites, fungi and bacteria, enhancing soil
fertility, reducing potential spread of diseases, and increasing site utility. Borrowing insects mix
organic and inorganic material, increasing soil porosity and water-holding capacity.
o Herbivores. Insects eating plants can cause substantial damage during their outbreaks. Yet it is
known that most plants can compensate for being eaten, and usually the weaker trees/plants are
eliminated. Besides, insects also produce very large amount of nutrient-rich material (insect frass,
molts, bodies and partially eaten foliage) that falls as litter. Nutrients that leach from this litter
stimulate turnover of organic material in the soil and enhance its health.
o Food source. Many birds feed their chicks with insects, which constitute a significant and
compulsory part of their protein diet. Major freshwater fisheries, especially those of salmonoids, are
supported largely or entirely by aquatic insects.
o Dispersal agents. Insects disperse seeds, transmit pathogenic agents, and even transport other
invertebrates from place to place (the phenomenon known as phoresy). Ants are amongst the most
important seed dispersers.
o Pollinators. Many plants depend on insects as pollinators of their flowers, and pollinators can be a
limiting resource for them.
 Hence option (d) is the correct answer.
 A few common insects whose existence is taken for granted and their ecological relevance are:
 Butterflies are important pollinators like bees. The lime butterfly is mostly dependent on citrus plant
species and other nectar plants throughout its life cycle and is a good indicator of environmental
health.
o Species diversity and density of butterfly indicate a good diversity of plants in an area. Climate
change, forest degradation, habitat loss, unavailability of hosts and nectar plant species are among
major reasons for a decline in butterfly population. This leads to loss of plants species that depend
on the butterflies for pollination. Farmers often use chemical insecticides to control lime
butterflies, causing them harm.
 Dragonflies, one of the most widely recognised insects, need clean aquatic systems and are hence a
good indicator of health of local aquatic systems. These, along with damsel flies, are well-known
biological predators with both larvae and adults acting as natural bio-control agents. They are highly
sensitive to changes in their habitats and are declining due to increasing habitat loss, anthropogenic
activities, pollutants, climate change and rapid urbanisation.
 Grasshoppers feed on different plants and can cause serious damage to economic crops. However, in
a biodiversity-rich region, they are an important component of the food chain, being an important
food source for many birds.
o Grasshoppers and insects such as crickets are often consumed by people as they rich in protein.
While monocropping systems can turn grasshoppers into pests, maintenance of plants can help
have enough grasshoppers to keep the ecosystem viable and healthy and retain bird biodiversity,
which would further help keep the numbers of grasshoppers under control.
 Ants are in the most abundance. Ants act as scavengers / decomposers by feeding on organic wastes
and other dead animals. Ants also aerate soil.
 Wild honey bees play a major role in pollination of forest species affecting cross pollination and
maintenance of variability within species. Wild honey is also a food source for humans and many wild
animals.
 Paper wasps are important for the environment. They act as predators and devour pests including
greenflies and caterpillars. Wasps are good pollinators as well.
 Rainbow leaf beetles are found in forests, woodlands and mountain grasslands. They mostly depend
on leaves and flowers of some specific plant family like Apocyanaceae. These are listed as
endangered species in International Union for Conservation of Nature from 1994.

Q 54.A
 The Round Table Conferences (RTC) were a series of three conferences conducted to deliberate upon and
bring about constitutional reforms in British India during 1930-32. There were three such conferences.
 The First Round Table Conference was held between November 1930 and January 1931 in London.

24 www.visionias.in ©Vision IAS


 The Second Round Table Conference was held in London from September 7, 1931, to December 1, 1931.
Besides the Congress, a large number of Indians participated in the Conference.
 Third and the last Round Table Conference was held in November – December 1932.
 Second Round Table Conference (RTC)
 The Indian National Congress nominated Gandhi as its sole representative. A. Rangaswami Iyengar and
Madan Mohan Malaviya were also there. Gandhiji sailed for London on 29 August 1931 to attend
the Second Round Table Conference (the only round table conference attended by Gandhiji) on the
basis of three principles of the federation, responsible government and reservation and safeguards. Apart
from a few able individuals, the overwhelming majority of Indian delegates to the Second Round Table
Conference, hand-picked by the Government, were loyalists, communalists, careerists, big landlords and
representatives of the princes. Hence statement 1 is correct.
 The second roundtable conference got deadlocked on the question of the minorities. Separate electorates
were being demanded by the Muslims, depressed classes, Christians and Anglo-Indians. All these came
together in a ‗Minorities‘ Pact‘. Gandhi fought desperately against this concerted move to make all
constitutional progress conditional on the solving of this Issue. Gandhi discarded the idea of a separate
electorate for untouchables. He also said there was no need for separate electorates or special safeguards
for Muslims or other minorities. Many of the other delegates disagreed with Gandhi.
 The lack of agreement among the many delegate groups in Second RTC led no substantial results
regarding India‘s constitutional future. The session ended with MacDonald‘s announcement of:
o two Mulsim majority provinces North-West Frontier Province (NWFP) and Sindh;
o the setting up of an Indian Consultative Committee;
o setting up of three expert committees—finance, franchise and states; and
o the prospect of a unilateral British Communal Award if Indians failed to agree.
 The government refused to concede the basic Indian demand for freedom. Gandhi returned to India on
December 28, 1931, and resume the Civil Disobedience Movement. Hence statement 2 is not correct.

Q 55.B
 Statement 1 is correct: Known collectively as the Bretton Woods Institutions after the remote village in
New Hampshire, U.S.A., where they were founded by the delegates of 44 nations in July 1944, the Bank
and the IMF are twin intergovernmental pillars supporting the structure of the world's economic and
financial order. The two institutions hold joint annual meetings, which the news media cover extensively.
Both have headquarters in Washington, D.C. The fundamental difference is this: the Bank is primarily a
development institution; the IMF is a cooperative institution that seeks to maintain an orderly system of
payments and receipts between nations. At Bretton Woods the international community assigned to the
World Bank the aims implied in its formal name, the International Bank for Reconstruction and
Development (IBRD), giving it primary responsibility for financing economic development.
 Statement 2 is not correct: The IMF provides financial support for balance of payments needs upon
request by its member countries. Unlike development banks, the IMF does not lend for specific projects.
Typically, a country's government and the IMF must agree on a program of economic policies before the
IMF provides lending to the country.
 Pakistan is seeking its largest loan package of up to USD 8 billion from the IMF to bail itself out from a
severe balance-of-payments crisis that threatens to cripple the country's economy.
 Statement 3 is not correct: Both the World Bank and IMF do not lend to individuals.

Q 56.A
 Torpor is a state of decreased physiological activity in an animal, usually by a reduced body
temperature and metabolic rate. Torpor enables animals to survive periods of reduced food availability.
To enter torpor, an animal decreases its metabolism, reducing its energy requirements. A torpid animal
will often be curled in a tight ball in its nest and look like it‘s sleeping. Hence statement 1 is correct.
 Once thought to occur only in birds and mammals in the Northern Hemisphere where winters are more
pronounced, we now know torpor is widespread in small Australian mammals and has also been
observed in many small Australian bird species. Hence statement 2 is not correct.
 Masters of metabolism: Birds and mammals are endotherms and can maintain a high and constant body
temperature independent of the environmental temperature, thanks to their high metabolic rate. This
allows them to be active across a wide range of environments.
 The downside?
 This high metabolic rate requires a lot of food to fuel it. By reducing the metabolism in a very controlled
manner and entering torpor, an animal can live on less energy.
25 www.visionias.in ©Vision IAS
 With a lower metabolic rate, the animal‘s body temperature decreases — sometimes by as much as 30°C.
How low it goes can depend on the extent of the metabolic reduction and the temperature of animal‘s
immediate environment. The reduced body temperature further lowers the metabolic rate.
 Slowing down to survive
 Torpor is an extremely effective survival strategy for small endotherms. For example, small mammals
have been observed using torpor after bushfires.
 Take the brown antechinus, for example. When other animals have fled, this 30 gram marsupial hides in
refuges, waits out the fire, then uses torpor to cope with reduced food availability until local vegetation
and invertebrate populations recover.
 Many pregnant and lactating bats and marsupials and even the echidna, synchronise torpor with
reproduction to cope with the energetic costs of mating, pregnancy or lactation.
 There are two main types of torpor: daily torpor and hibernation
 Daily torpor: Animals that use daily torpor can do so for approximately 3-6 hours a day as needed.
Daily torpor is common in, but not exclusive to, endotherms living in arid areas, such as the fat-
tailed dunnart. Fat-tailed dunnarts reduce their metabolic rate, and subsequently their body temperature,
from 35°C to approximately 15°C, or the temperature of their underground nest. Hence statement 3 is
correct.
 Hibernation: Animals that hibernate lower their metabolic rate further and have longer torpor bouts than
those that use daily torpor. An example of an Australian hibernator is the eastern pygmy possum, a
marsupial found in south-eastern Australia that hibernates regularly, decreasing its body temperature from
approximately 35 °C to as low as 5°C. When active, this species can survive for less than half a day on 1g
of fat, but when hibernating, it can survive for two weeks.
 Compare this with a well-known hibernator, the American black bear. At approximately 120kg, its
metabolic rate during hibernation decreases to 25 per cent of the basal metabolic rate and the body
temperature decreases from approximately 37°C to 30 °C. Black bears can‘t hibernate with a lower-body
temperature, perhaps because it would take them a very long time to reduce it and then cost them too
much energy to rewarm at the end of hibernation.

Q 57.D
 After independence, India launched its First Five Year Plan (FYP) in 1951, under the socialist influence of
first Prime Minister Jawaharlal Nehru. The process began with setting up of the Planning Commission in
March 1950 in pursuance of declared objectives of the Government to promote a rapid rise in the standard
of living of the people by efficient exploitation of the resources of the country, increasing production and
offering opportunities to all for employment in the service of the community.
 The Planning Commission was charged with the responsibility of making the assessment of all resources
of the country, augmenting deficient resources, formulating plans for the most effective and balanced
utilisation of resources and determining priorities.
 The first Five-year Plan was launched in 1951 and two subsequent five-year plans were formulated till
1965 when there was a break because of the Indo-Pakistan conflict. Two successive years of drought,
devaluation of the currency, a general rise in prices and erosion of resources disrupted the planning
process and after three Annual Plans between 1966 and 1969, the fourth Five-year plan was started in
1969.
 The fifth five-year plan was launched in the year 1974.
o It proposed to achieve two main objectives: 'removal of poverty' (Garibi Hatao) and 'attainment
of self-reliance. Hence option (d) is the correct answer.
o 'Promotion of high rate of growth, better distribution of income and significant growth in the
domestic rate of savings were seen as key instruments
o Due to high inflation, cost calculations for the Plan proved to be completely wrong and the original
public sector outlay had to be revised upwards. After promulgation of emergency in 1975, the
emphasis shifted to the implementation of Prime Ministers 20 Point Programme. FYP was relegated
to the background and when Janta Party came to power in 1978, the Plan was terminated.

Q 58.C
 India ranks 3rd worldwide for pharmaceutical production by volume and 14th by value. Since India
mostly produces Generic medicines which are comparatively cheaper i.e. low by value. Indian drugs
comprised 20 percent of the global generic drug exports, out of which North America had the largest
share.

26 www.visionias.in ©Vision IAS


 The country has an established domestic pharmaceutical industry, with a strong network of 3,000 drug
companies and ~10,500 manufacturing units.

 Himachal Pradesh has Asia‘s largest pharma manufacturing hub, is the Baddi-Barotiwala-Nalagarh
industrial belt, and the state produces around half of India‘s total drug formulations.
 Hence the correct option is (c)

Q 59.D
 Recently, the Pantanal Matogrossense National Park has been ravaged by months of forest fires.
 Pantanal Matogrossense National Park encompasses some 135,000 hectares across the municipality of
Pocone in the Brazilian state of Mato Grosso. The area has been under federal protection for nearly four
decades, with entry into the park strictly restricted in a bid to shield its rich ecological diversity.
 The park is a part of the Pantanal region, the world‘s largest tropical wetland, sprawling nearly
210,000 square kilometers (81,000 square miles) across Brazil, Bolivia and Paraguay. Hence option
(d) is the correct answer.
 It is home to the world‘s second-largest population of jaguars and provides habitat for dozens of species
threatened with extinction, like the hyacinth macaw
 Normally, the Pantanal is drenched by heavy rains between October and April, which flood about 80% of
the region, turning it into a maze of marshes and streams. But a prolonged drought has spelled disaster for
the region, with searing temperatures and high winds making the parched land highly susceptible to fires.

Q 60.B
 During the sixteenth and seventeenth centuries, about 85 per cent of the population of India lived in
villages. Both peasants and landed elites were involved in agricultural production and claimed rights to a
share of the produce.
 The basic unit of agricultural society was the village, inhabited by peasants who performed the manifold
seasonal tasks that made up agricultural production throughout the year. But rural India was not
characterised by settled peasant production alone.
 Sources of the seventeenth century refer to two kinds of peasants – khud-kashta and pahi-kashta. The
former were residents of the village in which they held their lands. The latter were non-resident cultivators
who belonged to some other village but cultivated lands elsewhere on a contractual basis. People became
pahi-kashta either out of choice – for example when terms of revenue in a distant village were more
favourable – or out of compulsion – for example, forced by economic distress after a famine. Hence,
statement 3 is correct.
 Monsoons remained the backbone of Indian agriculture, as they are even today. But there were crops
which required additional water. Artificial systems of irrigation had to be devised for this. Irrigation
projects received state support as well. For example, in northern India, the state undertook digging of new
canals (Nahr, Nala) and also repaired old ones like the shahnahr in Punjab during Shah Jahan‘s
reign. Hence, statement 2 is not correct.
 Agriculture was organised around two major seasonal cycles, the Kharif (autumn) and the rabi (spring).
The focus on the cultivation of basic staples did not mean that agriculture in medieval India was only for
subsistence. The Mughal state also encouraged peasants to cultivate jins-i Kamil (literally, perfect crops)
crops as they brought in more revenue. Crops such as cotton and sugarcane were jins-i Kamil par
excellence. Cotton was grown over a great swathe of territory spread over central India and the Deccan

27 www.visionias.in ©Vision IAS


plateau, whereas Bengal was famous for its sugar. Such cash crops would also include various sorts of
oilseeds (for example, mustard) and lentils. This shows how subsistence and commercial production were
closely intertwined in an average peasant‘s holding. Hence, statement 1 is correct.

Q 61.B
 Vertical farming is the practice of producing food on vertically inclined surfaces. Instead of farming
vegetables and other foods on a single level, such as in a field or a greenhouse, this method produces
foods in vertically stacked layers commonly integrated into other structures like a skyscraper, shipping
container or repurposed warehouse. Using Controlled Environment Agriculture (CEA) technology, this
modern idea uses indoor farming techniques. The artificial control of temperature, light, humidity,
and gases makes producing foods and medicine indoor possible. In many ways, vertical farming is
similar to greenhouses where metal reflectors and artificial lighting augment natural sunlight. The primary
goal of vertical farming is maximizing crops output in a limited space. . Hence statement 1 is not
correct. The unique geoponic (soil-based) platform is capable of growing over 200 types of crops that can
be planted and harvested easily, and grow to maturity within 21 days.
 Main characteristics of vertical farming are given below :
o It is one step higher than greenhouse technology and open field cultivation.
o It is being progressively practised in mushrooms, poultry, hydroponic fodder, strawberry, leafy-
vegetables particularly lettuce, herbs, ornamental horticulture and other crops production.
Hence statement 2 is correct.
o It is considered potential to provide sustainability to farming, combat chronic climate change and
reduce major inputs to crop production like land, water, nutrients, pesticides and others.
o It has the immense potential to attract the youth to farming by adding pride to the profession,
agriculture.
 Advantages:
o Ensures Consistent Crop Production
o Uses Space Optimally
o Reduces Usage of Water
o Cuts Down on Transport Cost
o Less Labour Costs
o Energy Efficient
o Doesn‘t Involve Chemicals or Pesticides
o Limits Occupational Hazards
o The Future of Farming
 Difficulties in vertical farming
o At present Vertical farming is performed only in a controlled, indoor environment. While this comes
with many advantages, it also hinders the entry or advent of insects. That means the process of
pollination is almost entirely hindered. In situations like these, cultivars need to consider manual
pollination. Hence pollination is difficult and costly.
o Huge initial cost for establishment.
o Higher labor costs as skilled workforce is required.
o It relies too much on of technology and any power loss could be disruptive and disastrous for the
produce.

Q 62.D
 Justice R. S. Sarkaria Commission was appointed in June 1983. It stressed on cooperative
federalism and noted that the federalism is more a functional arrangement for cooperative action
than a static institutional concept. At the same time, it did not equate strong centre with centralization
of powers because over-centralisation leads to blood pressure at the centre and anaemia at the periphery.
 M M Punchhi Commission was set up in 2007 and it gave its recommendations in 2010. Some of the
important recommendations are as follows:
o There should be a consultation process between union and states via Inter-state Council for legislation
on concurrent subjects.
o Regarding state bills, the President‘s pocket veto should end and there should be a reasonable time (6
months) in which President communicates his decision.
o The treaty making powers of union should be regulated and states should get greater participation in
treaties where interests of states are involved.
o Governor should get clear guidelines for appointment of Chief Ministers so that he does not mis-uses
his discretionary powers in this context. etc.
28 www.visionias.in ©Vision IAS
 Rajmannar committee was set in 1969. Apart from making a call for immediate constitution of Inter-
state Council, this committee made following recommendations:
o Union government should not take any decision without consulting the inter-state council when such
decision can affect the interests of one or more states.
o Every bill which affects interests of the states should be first referred to inter-state council before it is
introduced in parliament.
o Article 356 should be used only in rare cases of complete breakdown of law and order in state. etc.
 Hence all the committees made recommendations with respect to the Centre-State relations. Hence
option (d) is the correct answer.

Q 63.B
 The Fundamental Rights are enshrined in Part III of the Constitution from Articles 12 to 35. In this regard,
the framers of the Constitution derived inspiration from the Constitution of the USA (i.e., Bill of
Rights). Part III of the Constitution is rightly described as the Magna Carta of India.1It contains a very
long and comprehensive list of ‗justiciable‘ Fundamental Rights.
Category Consists of
(a) Equality before the law and equal protection of laws (Article
14).(b) Prohibition of discrimination on grounds of religion, race,
1. Right to equality (Articles 14- caste, sex or place of birth (Article 15).(c) Equality of opportunity in
18) matters of public employment (Article 16).(d) Abolition of
untouchability and the prohibition of its practice (Article 17).(e)
Abolition of titles except military and academic (Article 18).
(a) Protection of six rights regarding freedom of: (I) speech and
expression, (ii) assembly, (iii) association, (iv) movement, (v)
residence, and (vi) profession (Article 19).
2. Right (b) Protection in respect of conviction for offences (Article 20).(c)
to freedom (Articles 19-22) Protection of life and personal liberty (Article 21).(d) Right to
elementary education (Article 21A). Hence the correct option is
(b). (e) Protection against arrest and detention in certain
cases (Article 22).
(a) Prohibition of traffic in human beings and forced labour (Article
3. Right against exploitation
23).(b) Prohibition of employment of children in factories,
(Articles 23–24)
etc. (Article 24).
(a) Freedom of conscience and free profession, practice and
propagation of religion (Article 25).(b) Freedom to manage religious
4. Right to freedom of religion
affairs (Article 26).(c) Freedom from payment of taxes for promotion
(Article 25–28)
of any religion (Article 27).(d) Freedom from attending religious
instruction or worship in certain educational institutions (Article 28).
(a) Protection of language, script and culture of minorities (Article
5. Cultural and educational rights
29).(b) Right of minorities to establish and administer educational
(Articles 29–30)
institutions (Article 30).
Right to move the Supreme Court for the enforcement
6. Right to constitutional of fundamental rights including the writs of (i) habeas corpus, (ii)
remedies (Article 32) mandamus, (iii)prohibition, (iv) certiorari, and (v) quo war-rento
(Article 32).

Q 64.A
 Recent context: In order to tracing the covid-19 case, National Informatics Centre (NIC) under the
Ministry of Electronics & Information Technology (MeitY) developed the Aarogya Setu app.
 It is designed to keep track of other app users that a person came in contact with. It then alerts app users
if any of the contacts tests positive for COVID-19. Hence statement 1 is correct.
 The app uses the phone‘s Bluetooth and GPS capabilities. It will keep a record of all other Aarogya
Setu users that it detected nearby using Bluetooth. It will also use a GPS log of all the places that the
device had been at 15-minute intervals. It is available in multiple languages.
 Aarogya Setu app installation is not mandatory for a person wishing to travel by Air or Rail, its use
is voluntary. Hence statement 2 is not correct.
29 www.visionias.in ©Vision IAS
Q 65.C
 Jute is the second most important fibre crop of India after cotton. It is used for manufacturing gunny bags,
ropes, carpets, rugs, tarpaulins, etc. There was a great demand for jute because of its low price, softness
and strength.
 Conditions for Growth
o Jute crop requires a humid climate with temperature fluctuating between 24 degree Celsius and 38
degree Celsius.
o Minimum rainfall required for jute cultivation is 1000 mm with 80 to 90 per cent relative
humidity during the period of its growth.
o Sowing and raising of saplings are carried out in the pre-monsoon season with 25 cm to 55 cm of
rainfall. This is done to take full advantage of the monsoon season.
o The new grey alluvial soil of good depth receiving silt from annual floods is most suitable for
jute growth. However, jute is grown widely in sandy looms and clay loams.
o A large supply of cheap labour and a lot of water is necessary for processing the jute fibre post-
harvest.
 The leading world's jute producing countries are India, Bangladesh, China and Thailand. India is the
world's largest producer of raw jute and jute goods, contributing to over 50 per cent and 40 per cent
respectively of global production.
 Hence option (c) is correct.

Q 66.B
 The word Judicial activism is not defined in the constitution of India. The doctrine of judicial activism
was introduced in mid-1970s. Justice V.R. Krishna Iyer, Justice P.N. Bhagwati, Justice O. Chinnappa
Reddy and Justice D.A. Desai laid the foundations of judicial activism in the country. Hence statement 1
is not correct.
 In recent years law making has assumed new dimensions through judicial activism of the courts. The
judiciary has adopted a healthy trend of interpreting law in social context.
 Judicial activism describes judicial rulings suspected of being based on personal or political
considerations rather than on existing law. Sometimes judges appear to exceed their power in deciding
cases before the Court. They are supposed to exercise judgment in interpreting the law, according to the
Constitution. Judicial activists, however, seem to exercise their will to make law in response to legal
issues before the Court.
 The question of judicial activism is closely related to constitutional interpretation, statutory
construction and separation of powers. It is sometimes used as an antonym of judicial restraint.
 Judges should act more boldly when making decisions on cases:
o Law should be interpreted and applied based on ongoing changes in conditions and values.
o As society changes and their beliefs and values change, courts should then make decisions in cases the
reflect those changes.
 According to the idea of judicial activism, judges should use their powers to correct injustices,
especially when the other branches of government do not act to do so. In short, the courts should play
an active role in shaping social policy on such issues as civil rights, protection of individual rights,
political unfairness, and public morality.
 One of the criticism that the judiciary faces is that, in the name of judicial activism, the theory of
separation of powers is overthrown and the judiciary is undermining the authority of the legislature and
the executive by encroaching upon the spheres reserved for them. Hence statement 2 is not correct.
 Judicial activism has had mani-fold impact on the political system.
o It has democratised the judicial system by giving not just to individuals but also groups access to the
courts.
o It has enforced executive accountability. Hence statement 3 is correct.
o However, on the other hand, it has overburdened the courts because of too many Public Interest
Litigations being filed.

Q 67.D
 Recently India‘s External Debt: A Status Report: 2019-2020 was released. The following observations
were made in the report.
 India‘s external debt grew 2.8 per cent to US $ 558.5 billion as at end-March 2020. External debt as a
ratio to GDP rose marginally to 20.6 per cent as at end-March 2020 from 19.8 per cent a year ago. The
ratio of foreign currency reserves to external debt stood at 85.5 per cent as at end-March 2020.
30 www.visionias.in ©Vision IAS
 Statement 1 is not correct: Economic activity in India influences the accumulation of external debt,
reflecting the policy over the years of enabling the private sector to access foreign debt. Reflecting this, as
at end-March 2020, the stock of non-sovereign debt (private sector debt) is four times that of the
sovereign debt. Further, non-financial corporations are the biggest debtors, accounting for 42 per cent of
total debt, followed by deposit-taking corporations (28.3 per cent), and general government (18.1 per
cent).
 Statement 2 is not correct: Compared to end-March 2019, sovereign debt shrank 3 per cent to reach
US $ 100.9 billion. This decrease is primarily due to a decline in FII investment in G-Sec – the second
largest constituent – by 23.3 per cent to US $ 21.6 billion from US $ 28.3 billion a year ago. Loans from
multilateral and bilateral sources under external assistance– the largest constituent of the sovereign debt –
grew 4.9 per cent to US $ 87.2 billion.
 Non-sovereign debt, on the other hand, rose 4.2 per cent to US $ 457.7 billion mainly due to an
increase in commercial borrowings – the largest constituent – by 6.7 per cent to US $ 220.3
billion. Outstanding NRI deposits – the second largest constituent – at US $ 130.6 billion was
approximately equal to the level a year ago.
 Statement 3 is not correct: About 81 per cent of the total stock of external debt is long-term, i.e., having
maturity of greater than one year, predominately in the form of commercial borrowings and NRI deposits.
Remaining 19 per cent of the debt is short-term, primarily in the form of short-term trade credit. Short-
term trade credit, constituting about 95 per cent of the total short-term debt, is used for financing imports.
The US dollar is the predominant currency for the denomination of India‘s external debt with
a share of 53.7 per cent of the total debt as at end-March 2020. Hence option (d) is the correct
answer.

Q 68.B
 Bloodborne pathogens are microorganisms such as viruses or bacteria that are carried in blood and can
cause disease in people. There are many different bloodborne pathogens, including malaria, syphilis, and
brucellosis, and most notably Hepatitis B (HBV), Hepatitis C (HCV) and the Human
Immunodeficiency Virus (HIV). Hence options 2, 3 and 4 are correct.
 Bloodborne pathogens can be transmitted through contact with infected human blood and other potentially
infectious body fluids such as:
o semen
o vaginal secretions
o cerebrospinal fluid
o synovial fluid
o pleural fluid
o peritoneal fluid
o amniotic fluid
o saliva (in dental procedures), and
o any body fluid that is visibly contaminated with blood.
 The Hepatitis A virus was known to spread mainly through contaminated food and water and caused a
relatively milder form of liver inflammation. Hence, option 1 is not correct.

Q 69.B
 Government of India has notified a scheme for Viability Gap Funding to infrastructure projects that are to
be undertaken through Public-Private Partnerships. It is a Plan Scheme administered by the Ministry of
Finance with suitable budgetary provisions to be made in the Annual Plans on a year-to-year basis. the
fund is provided by Central Government only. Hence statement 1 is not correct.
 The earlier VGF scheme was limited to projects concerning economic infrastructure. The revamped VGF
scheme will attract more PPP projects and facilitate private investment in the social sectors (health,
education, wastewater, solid waste management and water supply, among others). The new scheme will
have two components: The sub-scheme-1 would cater to social sectors such as wastewater treatment,
water supply, solid waste management, health and education sectors, which often face bankability issues
on account of poor revenue streams. The sub scheme-2 will support demonstration or pilot social sectors
projects. The projects may be from health and education sectors where there is at least 50 per cent
operational cost recovery. Hence statement 2 is correct.

31 www.visionias.in ©Vision IAS


Q 70.A
 Indian Penal Code: It is the official criminal code of India. It is a comprehensive code intended to
cover all substantive aspects of criminal law.
o The code was drafted in 1860 on the recommendations of first law commission of India.
o The Indian Penal Code states various crimes and classifies them into multiple categories. The Code
also prescribes the penalties and the punishment for the respective crimes. Hence option 1 is
correct.
 CrPC is the main legislation on procedure for administration of substantive criminal law in India. It was
enacted in 1973, though initially created in 1882. It provides the machinery and procedure for the
investigation of crime, apprehension of suspected criminals, collection of evidence, determination of
guilt or innocence of the accused person and the determination of punishment of the guilty. Hence option
2 is not correct.
 The Criminal Procedure Code, 1973 provides for the powers of the courts and Magistrate, while the
Indian Penal Code does not. Hence option 3 is not correct.
 For example, if someone steals, then under section 379 of the IPC, he may be jailed for three years as
well as will be fined. That is, the full details of the determination of the sentence and the punishment of
the sentence is described in the IPC. Now the thief has stolen, but now there will be process to catch the
thief, i.e. how the criminal will be arrested, how the evidence of crime will be collected, how will bail be
granted, where the application for bail has to be applied, the accused determination of crime or innocence
are all jobs of the police, which come under the CRPC.

Q 71.D
 The CBI is headed by a Director and he is assisted by a special director or an additional director.
 The Director of CBI as Inspector-General of Police, Delhi Special Police Establishment (DSPE), is
responsible for the administration of the organisation.
 The Lokpal and Lokayuktas Act (2013) amended the DSPE Act (1946) according to which the Central
Government shall appoint the Director of CBI on the recommendation of a three-member
committee consisting of: (hence statement 1 is not correct)
o the Prime Minister as Chairperson,
o the Leader of Opposition in the Lok Sabha (where there is no recognized leader of opposition in the
Lok Sabha, then the leader of the single largest opposition party in the Lok Sabha would be a member
of that committee) and
o the Chief Justice of India or Judge of the Supreme Court nominated by him.
 The Director of CBI has been provided security of two-year tenure in office by the CVC Act, 2003.
 Section 4C of the DSPE Act provision clearly says that the CVC has no role, whatsoever, in curtailing
or extending the tenure of the CBI Director. Hence, statement 2 is not correct.

Q 72.A
 The Global Wage Report reviews the main trends in wages in developed, emerging and developing
countries. It also analyses inequality in the labour market and in household income. It is released by the
International Labour Organization. Hence option (a) is the correct answer.
 The International Labour Organization (ILO) is a United Nations agency whose mandate is to advance
social and economic justice through setting international labour standards. Founded in October 1919
under the League of Nations, it is the first and oldest specialised agency of the UN. The ILO has 187
member states: 186 out of 193 UN member states plus the Cook Islands. It is headquartered in Geneva,
Switzerland, with around 40 field offices around the world.
 ILO Global Wage Report – the seventh in the series – presents the emerging empirical evidence of the
effects of the crisis on wages. The ILO has estimated that the equivalent of 345 million full-time jobs
were lost in the third quarter of 2020. In spite of the promising development of vaccines, the crisis is far
from over.

Q 73.C
 Neutron stars are created when giant stars die in supernovas and their cores collapse, with the
protons and electrons essentially melting into each other to form neutrons.
 Neutron stars are city-size stellar objects with a mass of about 1.4 times that of the sun. Born from the
explosive death of another, larger stars, these tiny objects pack quite a punch.
 When stars four to eight times as massive as the sun explodes in a violent supernova, their outer layers can
blow off in an often-spectacular display, leaving behind a small, dense core that continues to collapse.
32 www.visionias.in ©Vision IAS
Gravity presses the material in on itself so tightly that protons and electrons combine to make neutrons,
yielding the name "neutron star."
 Neutron stars pack their mass inside a 20-kilometer (12.4 miles) diameter. They are so dense that a
single teaspoon would weigh a billion tons. On average, gravity on a neutron star is 2 billion times
stronger than gravity on Earth. In fact, it's strong enough to significantly bend radiation from the star in
a process known as gravitational lensing, allowing astronomers to see some of the backsides of the star.
 The power from the supernova that birthed it gives the star an extremely quick rotation, causing it to spin
several times in a second. Neutron stars can spin as fast as 43,000 times per minute, gradually slowing
over time.
 A quasar is an extremely luminous active galactic nucleus, in which a supermassive black hole with mass
ranging from millions to billions of times the mass of the Sun is surrounded by a gaseous accretion disk.
 Red Giant Stars: The next stage in a star‘s life happens when all the hydrogen in its core is used up and
the fusion stops. This means there is no longer an outward light pressure to counteract the inward pressure
pulling the star together. The life of the star continues when the shell of hydrogen around the core ignites
and causes the star to massively increase in size. The star then becomes a red giant and can be up to 100
times bigger than it was in its main-sequence stage. This stage of stellar evolution only lasts a few
hundred million years. Its life-span is so short because the star only has so many layers of helium to
consume before it runs out and becomes a white dwarf.
 White Dwarf: A white dwarf is what stars like the Sun become after they have exhausted their nuclear
fuel.
 A black hole is not an empty space but a place in space where gravity pulls so much that even light can
not get out. The gravity is so strong because the matter has been squeezed into a tiny space. This can
happen when a star is dying. Black holes can be big or small. Hence option (c) is the correct answer.

Q 74.B
 Recent Context: Now, the government is choosing for direct benefit transfer (DBT) of urea subsidy to
the beneficiary farmers‘ bank accounts instead of DBT to firms based on point of sale.
o This move will reduce the leakage of fertilizer subsidy and black marketing.
o The ceiling might be put on the subsidized fertilizer so that the alleged overuse of the nitrogenous
fertilizer could be curbed.
o Earlier, mandatory neem-coated urea production was done to slow down the dissolution of nitrogen
into the soil, resulting in less nutrient requirement.
 Nutrient Based Subsidy scheme:
o Under this, the government announces a fixed rate of subsidy (in Rs. per Kg basis), on each
nutrient of subsidized P&K fertilizers, namely Nitrogen (N), Phosphate (P), Potash (K) and Sulphur
(S).
o It is applicable to 22 fertilizers (other than Urea) for which MRP will be decided to take into
account the international and domestic prices of P&K fertilizers, exchange rate, and inventory
level in the country.
o Urea Subsidy is a part of Central Sector Scheme of Department of Fertilizers and is wholly
financed by the Government of India through Budgetary Support.
o The New Urea Policy-2015 (NUP-2015) has been notified by Department of Fertilizers in 2015,
extended till 2019-2020, with the objective of maximizing indigenous urea production, promoting
energy efficiency in urea production and rationalizing subsidy burden on the government.
 It is applicable to the existing 25 gas-based units.
 The continuation of Urea Subsidy Scheme till 2020 will ensure the timely payment of subsidy to
the urea manufacturers resulting in timely availability of urea to farmers.
 Subsidy on production costs is provided when their production is beyond a certain production
capacity as notified.
 From March 2018, a new direct benefit transfer (DBT) system was introduced.
o In this, the subsidy is given directly to farmers. Here the manufacturer sells Urea at the market-
based price, the farmer will have to pay entire market price at the time of purchase and she/he will get
back this amount from the Government as DBT.
o Each retailer has a point-of-sale (PoS) machine linked to the Department of Fertilisers‘ e-
Urvarak DBT portal. Anybody buying subsidised fertilisers is required to furnish their Aadhaar or
Kisan Credit Card (KCC) number.
o Only upon the sale getting registered on the e-Urvarak platform can a company claim subsidy, with
these being processed on a weekly basis and payments remitted electronically to its bank account.

33 www.visionias.in ©Vision IAS


 How does the subsidy system work for urea and non-urea fertilizer?
o The Centre controls the Maximum Retail Price (MRP) of urea, unrelated to the cost of production
and distribution, which is higher. It reimburses the manufacturers for the shortfall as subsidy on
a ―unit-specific‖ basis under the New Pricing Scheme (NPS).
o The current MRP of urea is low at Rs 5,360 per tonne even as the actual cost can be as high as Rs
25,000 per tonne. This means the manufacturer gets a subsidy of Rs 19,640 per tonne.
o In the case of non-urea fertilizers like phosphorus (P) and potassium (K) fertilisers, the
government decontrolled the prices in 1992 and introduced Nutrient Based Subsidy Policy in
2010.
o This means the Government gives ―uniform‖ subsidy on per nutrient basis to all manufacturers
under the Nutrient-Based Subsidy (NBS) Scheme. Hence statement 1 is not correct.
 Issues in the subsidy system
o Availability: Since the sale of urea is controlled, the government needs to estimate demand in each of
the regions. Inaccurate estimation of demand of urea had led to large shortages in the market. o
Delays in imports also have led to unavailability of fertilizer around planting seasons when the need is
most critical.
o Over usage/misuse of urea due to pricing difference: Growing price differential between urea and
other fertilizers led farmers to substitute away from di-ammonium phosphate (DAP) and muriate of
potash (MOP) to urea. This disincentivizes the farmer from using urea in excess of his/her
requirements and in long run will correct the distorted consumption N:P: K equilibrium. Hence
statement 2 is correct.
 Data from the Department of Agriculture shows that since 2010, the ratio of consumption has
worsened to 8:3:1 leading to diminishing crop yields and increased soil toxicity.
o Inefficient Fertiliser Manufacturers: The subsidy a firm receives is based on its cost of production:
the greater the cost, the larger the subsidy. As a consequence, inefficient firms with high production
costs survive and the incentive to lower costs is blunted.
o The bulk of the applied urea is lost as ammonia (NH3), dinitrogen (N2) and NOx (nitrogen
oxides). The released ammonia gets converted to nitrates, increasing soil acidity, NOx gases are major
air pollutants. Nitrate contamination of groundwater leads to conditions such as
methaemoglobinaemia (commonly known as blue baby syndrome) and this has reached far beyond
WHO safe limit in states like Punjab, Haryana and Rajasthan. Hence statement 3 is correct

Q 75.B
 Article 22 grants protection to persons who are arrested or detained. Detention is of two types, namely,
punitive and preventive. Punitive detention is to punish a person for an offence committed by him after
trial and conviction in a court. Preventive detention, on the other hand, means detention of a person
without trial and conviction by a court. Its purpose is not to punish a person for a past offence but to
prevent him from committing an offence in the near future. Thus, preventive detention is only a
precautionary measure and based on suspicion.
 The Article 22 has two parts—the first part deals with the cases of ordinary law and the second part deals
with the cases of preventive detention law.
 The first part of Article 22 confers the following rights on a person who is arrested or detained under an
ordinary law:
o Right to be informed of the grounds of arrest. Hence statement 3 is correct.
o Right to consult and be defended by a legal practitioner. Hence statement 2 is correct.
o Right to be produced before a magistrate within 24 hours, excluding the journey time.
o Right to be released after 24 hours unless the magistrate authorises further detention. Hence
statement 1 is not correct.
 These safeguards are not available to an enemy alien or a person arrested or detained under a preventive
detention law.
 The second part of Article 22 grants protection to persons who are arrested or detained under a
preventive detention law. This protection is available to both citizens as well as aliens and includes the
following:
o The detention of a person cannot exceed three months unless an advisory board reports sufficient
cause for extended detention. The board is to consist of judges of a high court.
o The grounds of detention should be communicated to the detenu. However, the facts considered to be
against the public interest need not be disclosed.
o The detenu should be afforded an opportunity to make a representation against the detention order.

34 www.visionias.in ©Vision IAS


Q 76.C
 The word Shramana means one who performs the act of austerity and ascetic. It refers to several Indian
religious movements parallel to Vedic religion. The various Shramana schools include Jainism,
Buddhism, Ajivikas, Ajnanas, Charvakas. Hence statement 1 is correct.
 All five mentioned above belong to the Nastika or Heterodox school of philosophy.
 Ajivika school
o The school was founded by Makkhali Gosala in 5th century BC. The school revolves around the
Niyati (Fate) doctrine of absolute determinism. It believes that there is no free will and whatever has
happened, is happening or will happen is entirely pre-ordained or predecided and is based on cosmic
principles. Hence there was no use of Karma.
o They opposed Buddhism and Jainism. They did not believe in the Karma doctrine, unlike Jainism
and Buddhism. They consider Karma a fallacy. Hence statement 2 is not correct.
o They also rejected the authority of Vedas like Buddhism and Jainism.
 Ajnanas
o The ajnana sect believed in radical scepticism.
o The school believed that it is impossible to attain knowledge about nature. Even if it is possible,
it is useless for attaining salvation. Hence statement 3 is correct.
o This school was a major rival of Jainism and Buddhism.
o They specialised in refutation and were considered ignorant.
o They believed that ―Ignorance is Best‖.

Q 77.A
 A single wheel cannot turn‖, says the Arthashastra, rather inaccurately, ―and so the government is only
possible with assistance. Therefore a king should appoint councillors and listen to their advice. ‖ At the
head of affairs was a small body of elder states¬ men, whom the king was advised to choose with the
utmost care. In fact, the council often exerted great powers. It might transact business in the king‘s
absence, and the Asokan inscriptions show that it might make minor decisions without consulting him.
 The functions of the councillors were not always sharply defined or delimited, and terminology varied
considerably. There seems usually to have been a chief counsellor, the mantrin par excellence, often
called ―great counsellor‖ (mahamantrin). With orthodox kings, the purohita or court chaplain was very
influential, and one source even suggests that before coming to a final decision the king should deliberate
privately with him.
 The treasurer and chief tax- collector (called in the Arthasastra sannidhatr and samahartr respectively),
were important, as was the ―minister of peace and war‖ {sandhivigrahika), a title which does not
appear until Gupta times. Hence, pair 1 is correctly matched.
 This minister approximated to the foreign secretary of the modern state, but had more definite military
functions and often accompanied the king on a campaign. The pradvivaka, or chief judge and legal
advisor, seems to have been important in medieval Hindu kingdoms, and the Senapati, or general, was
always influential, while the mahaksapatalika, or chief record keeper and secretary, no doubt attended
the council meetings. Hence, both pairs 2 and 3 are not correctly matched.

Q 78.B
 Recently, India backed a UK-led campaign against end-to-end encryption (E2EE) of messages by social
media giants. The UK, India, US, Canada, Australia, New Zealand and Japan in a joint international
statement addressed to all tech companies to ensure they do not blind themselves to illegal activity on
their platforms, including child abuse images.
 End-to-end encryption is the act of applying encryption to messages on one device such that only
the device to which it is sent can decrypt it. The message travels all the way from the sender to the
recipient in encrypted form. Hence statement 1 is not correct.
 The other alternative for the transfer of data is in clear text, that is, without encrypting the message at all.
That is the least secure option. For example, data sent by SMS is not encrypted, meaning that in theory,
anyone can intercept it. Fortunately, in practice, doing so requires special equipment, which somewhat
limits who can eavesdrop on your text messages. Hence statement 2 is correct.
 Another option for the transfer of data is encryption-in-transit, whereby messages are encrypted on
the sender‘s end, delivered to the server, decrypted there, re-encrypted, and then delivered to the recipient
and decrypted on their end. Encryption-in-transit protects information during transmission, but
using it allows the intermediate link in the chain — the server — to see the content. Depending on
how trustworthy its owners are, that can be an issue.

35 www.visionias.in ©Vision IAS


 At the same time, using encryption-in-transit includes the server in the communication, which opens up a
range of services that go beyond simple data transfer. For example, a server can store message history,
connect additional participants using alternative channels to a conversation (such as joining a video
conference by phone), use automatic moderation, and more.
 People prefer end-to-end encrypted messaging on various apps because it keeps their messages safe from
hackers, criminals, and foreign interference.
Q 79.C
 The Attorney General (AG) is appointed by the president. He must be a person who is qualified to be
appointed a judge of the Supreme Court. Hence, statement 1 is correct.
 The term of office and remuneration of the AG is not fixed by the Constitution.
 Further, the constitution does not contain the procedure and grounds for his removal.
 Hence, he holds office during the pleasure of the president. Hence, statement 2 is correct.
 Conventionally, he resigns (addressing to President) when the government resigns or is replaced, as he is
appointed on its advice.
 Attorney General has the right of audience in all courts in the territory of India.
 Further, he has the right to speak and to take part in the proceedings of both the Houses of Parliament
or their joint sitting and any committee of the Parliament of which he may be named a member,
but without a right to vote. Hence, statement 3 is not correct.
 He enjoys all the privileges and immunities that are available to a member of Parliament.

Q 80.A
 The Global Atmosphere Watch (GAW) is a worldwide system established by the World
Meteorological Organization – a United Nations agency – to monitor trends in the Earth's atmosphere. It
arose out of concerns for the state of the atmosphere in the 1960s.
 The Global Atmosphere Watch (GAW) Programme of WMO focuses on building a single coordinated
global understanding of atmospheric composition, its change, and helps to improve the understanding of
interactions between the atmosphere, the oceans and the biosphere. About 100 countries are
participating in the GAW Programme. Some components of the GAW observational network are
recognized as comprehensive and baseline networks of the Global Climate Observing System (GCOS).
The GAW Programme operates according to the GAW Implementation Plan 2016-2023.
 One major aspect of the GAW mission is to organize, participate in and coordinate assessments of the
chemical composition of the atmosphere on a global scale. In this way, GAW provides reliable scientific
information for policymakers, supports international conventions and contributes to improve the
understanding of climate change and long-range transboundary air pollution.
 Earth Overshoot Day marks the date when humanity‘s demand for ecological resources and services in a
given year exceeds what Earth can regenerate in that year. It is declared every year by the Global
Footprint Network.
 Hence option (a) is the correct answer.

Q 81.A
 Sir Syed Ahmed Khan was renowned Muslim reformer of the 19th century.
 He was a great educationist and a social reformer but a conservative politician.
 Sir Syed stressed on the importance of modern scientific education for Muslims to advance their
conditions. Hence, statement 2 is not correct.
 He advocated the learning of English. He was also against superstition and evil customs prevalent in
society then.
 He set up many educational institutes to propagate education, the most significant being
the Muhammadan Anglo-Oriental College (MAOC) which he set up in 1875. This later became the
Aligarh Muslim University.
 Syed Ahmed loudly preached the commonness of Hindus and Muslims till the founding of the Congress
in 1885. But then, he opposed the formation of INC. Hence, statement 3 is correct.
 In the 1880's he gave up his earlier views and declared that the political interests of Hindus and Muslims
are not the same. He preached Muslims, the complete obedience to British rule. Hence, statement 1
is correct.
 Initially, he made an attempt with the help of Shiva Prasad, Raja of Bhinga, and others to organize a
loyalist movement against the rising democratic national movement.
 He is regarded as one of the founders of the Two-Nation Theory which says that Hindus and Muslims
cannot be one nation.
36 www.visionias.in ©Vision IAS
Q 82.C
 Recent Context: The massive reverse migration of people from urban areas to villages ever since
lockdown was imposed due to COVID-19 has put focus on the Attracting and Retaining Youth in
Agriculture (ARYA) project of the Centre with several experts suggesting that time is now ideal to go
for full-fledged implementation of the ambitious scheme.
 Realizing the importance of rural youth in agricultural development of the country, Indian Council of
Agricultural Research (ICAR), Ministry of Agriculture & Farmers Welfare has initiated ARYA
project.
 The objectives of ARYA scheme are:
o To attract and empower the Youth in Rural Areas to take up various Agriculture, allied and service
sector enterprises for sustainable income and gainful employment in selected districts.
o To enable the farm youth to establish network groups to take up resource and capital-intensive
activities like processing, value addition and marketing. However there is no such provision for
minimum percentage of participation of women under the scheme. Hence statements 1 and
statement 2 are not correct.
o ARYA project will be implemented in 25 States through ICAR-Krishi Vigyan Kendra
(KVKs), one district from each State. In one district, 200-300 rural youths will be identified for
their skill development in entrepreneurial activities and establishment of related micro-enterprise
units. Hence statement 3 is correct.
o KVKs will involve the Agricultural Universities and ICAR Institutes as Technology Partners. At
KVKs also one or two enterprise units will be established so that they serve as entrepreneurial training
units for farmers.

Q 83.D
 The Cabinet Secretariat functions directly under the Prime Minister.
 The administrative head of the Secretariat is the Cabinet Secretary. She/He is appointed by the
Union Government. Hence statement 1 is not correct.
 She/He presides over the Annual conference of Chief Secretaries. Hence statement 3 is correct.
 About Cabinet Secretariat:
o Cabinet Secretary is the ex-officio Chairman of the Civil Services Board. Hence statement 2 is
correct.
o The business allocated to Cabinet Secretariat under Government of India (Allocation of Business)
Rules, 1961 includes:
 Secretarial assistance to the Cabinet and Cabinet Committees; and
 Rules of Business
o The Cabinet Secretariat is responsible for the administration of the Government of India
(Transaction of Business) Rules, 1961 and Government of India (Allocation of Business) Rules,
1961 facilitating smooth transaction of business in Ministries/ Departments.
o The Secretariat assists in decision-making in Government by ensuring Inter-Ministerial coordination,
ironing out differences amongst Ministries/Departments and evolving consensus through the
instrumentality of the standing/adhoc Committees of Secretaries.
o Management of major crisis situations in the country and coordinating activities of various ministries
in such a situation is also one of the functions of the Cabinet Secretariat.

Q 84.C
 Centre for Monitoring Indian Economy (CMIE) reported that India‘s unemployment rate has eased to
pre-lockdown levels of 8.5 per cent by June 2020.
 Unemployment rate is the percent of the labor force that is without work i.e. Unemployment rate =
(Unemployed Workers / Total labor force) X 100.
 An unemployed worker is someone who is part of labour force and not engaged in economic activity
(work) but making tangible efforts to seek work or is available for work if the work is available.
 This means Unemployment Rate does not include that section of the population who is not seeking work
or available for work.
 Another related measure is Employment to population ratio (Epop) also called as Worker Population
Ratio (WPR).
 WPR is defined as the proportion of an economy‘s working-age population that is employed and is
expressed as WPR= No. of employed persons x 1000 /Total population.

37 www.visionias.in ©Vision IAS


 Coming back to the question, a situation of low unemployment rate and high unemployed workers could
be because the workers who lost jobs during the pandemic were discouraged from even looking for
jobs, and left the labour force entirely.
 Thus the proper explanation to such a situation is that while the unemployment rate bounced back,
India‘s Epop or worker population ratio has not fully recovered which is reflected in the WPR data.
WPR is about 2 percentage points below pre-Covid levels. This suggests that India has 20-40 million
fewer employed people than it did before Covid. Hence option (c) is the correct answer.
 This also means that even if those people are not looking for jobs today, they might start looking for jobs
as the country reopens. This could cause a surge in the unemployment rate in the months to come.

Q 85.C
 TRIFED is training local tribals in Barwani, Madhya Pradesh in the Bagh, Maheshwari and Chanderi
textile crafts.
 Bagh Print is a traditional hand block print with natural colours, practised in Dhar district in Madhya
Pradesh. Bagh printing received the Geographical Indication (GI) tag in 2008. Hence pair 1 is not
correctly matched.
 The Tanchoi style of weaving which resembles fine miniatures owes its origin to China and practised
mainly in Surat at Gujarat. The Tanchoi saris are very popular. Hence pair 2 is not correctly matched.
 Bagru is a small town in Jaipur Rajasthan. Bagru is known for natural dyes and hand block
printing. Hence pair 3 is correctly matched. Bagru hand block print also received the GI tag.

Q 86.D
 India‘s second rocket launch pad is being set up in Thoothukudi (formerly known as Tuticorin)
district in Tamil Nadu. The project will house one launchpad exclusively for small satellite launch
vehicles (SSLV).
 India presently has one rocket port at Sriharikota in Andhra Pradesh with two launch pads.
 Why was Thoothukudi chosen?
o Geostrategic location: In polar missions, the PSLV from Sriharikota must perform a dogleg
manoeuvre (deviation of a rocket from the straight flight path) to avoid flying over Sri Lanka, to
protect it from rocket debris.
o Rockets launched from Thoothukudi don‘t require this manoeuvre as there is no landmass
along the flight path in the southward direction. It will also save the rocket‘s fuel as well as
improve the payload capability.
o Proximity to critical infrastructure: Thoothukudi is around 70-100 km away from ISRO‘s Liquid
Propulsion Systems Centre (LPSC) in Tirunelveli. This will save time and cost of transportation as
compared to Sriharikota launch pad.
o Proximity to Earth‘s equator: India prefers its spaceports as close to the equator as possible and
located on the east coast for two reasons:
o Earth‘s rotation provides a speed boost to rockets launched and strength of the boost is higher closer
to the equator.
o In the event of a failure, debris from an explosion would fall into the Bay of Bengal instead of land,
potentially saving property and lives. Hence option (d) is the correct answer.

Q 87.B
 The Lok Adalat is an innovative Indian contribution to the global jurisprudence. It is one of the efficient
alternative dispute resolution mechanisms that have the potential to provide amicable settlements of
differences. It also provides for an inclusive justice as envisaged by the Constitution of India.
 This system, based on Gandhian principles, aims to settle disputes through arbitration at the grass-root
level.
 Permanent Lok Adalats:
o These were given the statutory status under the Legal Services Authorities Act, 1987. Hence
statement 2 is correct.
o These have been set up as permanent bodies with a Chairman and two members for providing
compulsory pre-litigative mechanism for conciliation and settlement of cases relating to Public Utility
Services like transport, postal, telegraph etc.
o A person who is, or has been, a district judge or additional district judge or has held judicial
office higher in rank than that of a District Judge, shall be entitled to be a Chairman of the
Permanent Lok Adalat. Hence statement 1 is not correct.

38 www.visionias.in ©Vision IAS


o Here, even if the parties fail to reach to a settlement, the Permanent Lok Adalat gets jurisdiction to
decide the dispute, provided, the dispute does not relate to any offence. Further, the Award of the
Permanent Lok Adalat is final and binding on all the parties.
o The jurisdiction of the Permanent Lok Adalats is upto Rs. One crores.

Q 88.D
 Commensalism
 Commensalism is a type of symbiosis in which spatial proximity allows the commensal to feed on
substances captured or ingested by the host. The two partners can survive independently. In
commensalism, one organism benefits while the other is unaffected. For example, one organism can
provide an essential growth factor, such as a vitamin, for another organism. An example of commensalism
is the association of hermit crabs and the sea anemones they carry on their borrowed shells.
 Examples of Commensalism
 Remora fish have a disk on their heads that makes them able to attach to larger animals, such as sharks,
mantas, and whales. When the larger animal feeds, the remora detaches itself to eat the extra food.
 Nurse plants are larger plants that offer protection to seedlings from the weather and herbivores, giving
them an opportunity to grow.
 Mites may be the ultimate commensals. These tiny arachnids live on the bodies or in the nests of
thousands of species (including humans!). While some mites are parasitic, most are far too small and
passive to have any effect.
 Tree frogs use plants as protection.
 Golden jackals, once they have been expelled from a pack, will trail a tiger to feed on the remains of its
kills.
 Goby fish live on other sea animals, changing colour to blend in with the host, thus gaining protection
from predators.
 Cattle egrets eat the insects stirred up by cattle when they are grazing. The cattle are unaffected, while
the birds gain food.
 The burdock plant produces spiny seeds that cling to the fur of animals or clothing of humans. The
plants rely on this method of seed dispersal for reproduction, while the animals are unaffected.
 Epiphytic plants are plants that grow on support, often another plant, to reach air, water and sunlight.
Some are parasites, like the accurately named strangler fig. Others, including species of orchid and fern,
don't affect the plant they're balanced on.
 Association of hermit crabs and the sea anemones they carry on their borrowed shells.
 Tree frogs use plants as protection. Hence option (d) is the correct answer.

Q 89.D
 In India, total Central Government Liabilities constitutes the following three categories :
o Internal Debt.
o External Debt.
o Public Account Liabilities
 Public Debt in India includes only Internal and External Debt incurred by the Central Government.
Hence, statement 1 is not correct.
 Internal Debt includes liabilities incurred by resident units in the Indian economy to other resident units,
while External Debt includes liabilities incurred by residents to non-residents.
 The major instruments covered under Internal Debt are as follows:
o Dated Securities
o Zero-coupon securities
o 14 Day Treasury Bills.
o Securities issued to International Financial Institutions
o Securities issued against ‗Small Savings‘: All deposits under small savings schemes are credited to
the National Small Savings Fund (NSSF).
o Market Stabilization Scheme (MSS) Bonds
 Public Debt Management Agency (PDMA) is a specialized independent agency that manages the
internal and external liabilities of the Central Government in a holistic manner and advises on such
matters in return for a fee.
 In an effort to streamline the borrowings of the government and achieve better cash management, the
Finance Ministry in 2016 established the Public Debt Management Cell which is to become Public Debt
Management Agency in two years time. But the creation of PDMA is put on hold as Creation of a
39 www.visionias.in ©Vision IAS
PDMA was a matter of intense debate in India. Many, at some phase even RBI, believed that debt
management functions should be continued with RBI. Hence, as of now, RBI is managing government
debt including public debt. Hence statement 2 is not correct.

Q 90.A
 Recently, a medicane named Ianos made landfall along the coast of Greece.
 Medicanes (MEDIterranean hurriCANES) refer to tropical stormlike cyclone observed across the
Mediterranean Sea. Hence the correct option is (a).
o Medicanes occur more in colder waters than tropical cyclones, hurricanes and typhoons. Hence, the
cores of these storms are colder in comparison to the warm cores of tropical cyclones.
o Since warmer cores tend to carry more moisture (hence rainfall), are bigger in size, and have swifter
winds, medicanes are weaker and smaller in size than the tropical cyclones.
o Like tropical storms, medicanes have a symmetric structure and a clearly visible eye.
 The Mediterranean is a generally dry, evaporative sea and cyclonic storms don‘t grow as much.
According to a study published in 2011, only one or two medicanes occur per year.
 However, recent studies have shown that medicanes are likely to become a bigger problem as the planet
warms due to climate change.
o Warmer sea surface temperatures in the Mediterranean can allow the storms to take on more tropical
appearances and characteristics,increasing the wind speeds and making the storms more intense with
stronger winds and heavier rainfall.
 Impact of La Nina: La Niña is characterized by the unusual cooling of the central and east-central
equatorial Pacific Ocean.
o A La Niña produces more rain in the central-eastern part, where most of the Mediterranean cyclones
develop. The slopes and the convection rising from sea waters can combine to spin off these cyclonic
storms that become a Medicane.
 A Japanese Space Mission Hayabusa2 has brought a piece of Asteroid Ryugu down to Earth recently.
 The Union Health Ministry has developed a digital platform, including an application CoWin, for real-
time monitoring of Covid-19 vaccine delivery, recording data and to enable people to get themselves
registered for vaccination.

Q 91.C
 Recently, the research team at S. N. Bose National Centre for Basic Sciences, Kolkata, an autonomous
institute of the Department of Science & Technology, Government of India, has spotted a new biomarker
called ‗BreathPrint‘ for diagnosis of Helicobacter pylori in semi-heavy water (HDO) in human
exhaled breath.
 The team has used ‗study of different water molecular species in human breath, also called ‗Breathomics‘
method to explore different water isotopes in human exhaled breath.
 Exhaled breath may soon help detect bacteria that infect the stomach, causing various forms of
gastritis and eventually gastric cancer. Scientists have found a method for early diagnosis of
bacteria that causes peptic ulcer, with the help of a biomarker called ‗BreathPrint‘ found in the
breath. Hence option (c) is the correct answer.
 Helicobacter pylori, a common infection which may turn serious if it is not treated early, is usually
diagnosed by the traditional and invasive painful endoscopy and biopsy tests which are not suitable for
early diagnosis and follow up.
 Our gastrointestinal (GI) track plays a crucial role in water metabolism in the body. Water exists in
nature as four isotopes. It is believed that any kind of impaired or unusual water absorption in our GI
track may be associated with various gastric disorders or abnormalities like ulcer, gastritis, erosions and
inflammation. But so far there has been no clear experimental evidence to support this.
 Experiments by the team has shown direct evidence of unique isotope-specific water-metabolism in the
human body in response to the individual‘s water intake habit. They have shown that the different
isotopes of exhaled water vapour are strongly linked with various gastric disorders during the
process of human respiration.

Q 92.C
 Hojagiri: Hojagiri is a folk dance, performed in the state of Tripura by the Reang people. The dance
is performed by women and young girls, about 4 to 6 members in a team, singing, balancing on an earthen
pitcher and managing other props such as a bottle on the head and earthen lamp on the hand, while only

40 www.visionias.in ©Vision IAS


the lower half of the body is moved. It is attractive due to its melodious songs accompanied by bamboo
flute, cymbals and the khamb. Hence pair 1 is not correctly matched.
 Cheraw dance is performed in the north eastern state of Mizoram. This dance includes four people. It
is the most notable dance of Mizoram and a center for attraction during festive season in Mizoram.
Similar dance forms could be found in East and even in Philippines (known as Tinikling.) Hence pair 2 is
correctly matched.
 Kamsale dance: The Kamsale dance is named after the instrument held in the hands of the dancer. The
Kamsale artistes or dancers are found in the Kannada speaking areas of Mysore, Nanjangudu,
Kollegal and Bangalore. The instrument comprises a cymbal held in one hand and a bronze disc in the
other. The main element in art is the rhythmic clang, which blends with the melodious music of the
Mahadeeshvara epic. The instruments, in the course of the vigorous rhythmic beatings, are moved around
the body of the dancer in innumerable patterns manifesting both skill and art. In a group movement, the
dancer provides the vision of a series of offensive and defensive manoeuvres. Hence pair 3 is correctly
matched.

Q 93.B
 Nominated members in Rajya Sabha:
o Under Article 80 of the Constitution, the Council of States (Rajya Sabha) is composed of not more
than 250 members, of whom 12 are nominated by the President of India from amongst persons who
have special knowledge or practical experience in respect of such matters as literature, science, art
and social service.
o By adopting the principle of nomination in Rajya Sabha, the Constitution has ensured that the nation
must also receive services of the most distinguished persons of the country who have earned
distinction in their field of activity, many of whom may not like to face the rough and tumble of the
election.
o By nominating them to Rajya Sabha, the State not only recognises their merit and confers honour on
them, but also enables them to enrich the debates by their expertise and knowledge that they have in
different areas.
o -Since the Rajya Sabha was constituted in 1952, in all, one hundred and five members have so far
been nominated to the House. Smt. Rukmini Devi Arundale (1952-56 and 1956-62) was the first
woman member nominated to the Rajya Sabha.
 Nominated members enjoy all powers, privileges and immunities available to an elected member of
Parliament.
o They take part in the proceedings of the House as any other member.
o They, however, are not entitled to vote in the election of the President of India. But in the election
of the Vice-President of India, they have a right to vote. Hence statement 1 is not correct.
o A nominated member is allowed six months, should he decide to join a political party after he
has taken his seat in the House in terms of Article 99 of the Constitution. A nominated member has
also been exempted from filing his assets and liabilities under Section 75A of the Representation of
the Peoples Act, 1951 which requires the elected member to do so within 90 days of his making or
subscribing oath/affirmation. Hence statement 2 is correct.
 The nominated members of either House of Parliament can participate in the impeachment of the
President though they do not participate in his election. Hence statement 3 is correct.

Q 94.D
 Recent context – All India Trade Union Congress (AITUC), formed in 1920 has turned 100 years of
existence.
 About AITUC
o Trade Union movement began in India after the end of First World War. After a decade following the
end of the First World War, the pressing need for the coordination of the activities of the individual
unions was recognised.
 The first organized Trade Union in India named as the Madras Labour Union was formed in the year
1918. It was formed by B.P Wadia. Hence statement 1 is not correct.
 On October 31, 1920, first All India Trade Union Congress (AITUC) was formed. It began to hold its
sessions every year from 1920 at different centres.
 The first session of AITUC was held in the Empire Theatre, Bombay. In the first conference, Lala Lajpat
Rai as elected president and V. M.Pawar as the first General Secretary (and not Diwan Chamanlal). It
was attended by political leaders of various shades of opinions such as Moti Lal Nehru, Mohd. Ali Jinnah,

41 www.visionias.in ©Vision IAS


Annie Basant, V J Patel, B.P. Wadia, J. Baptista, Lalubhai Samaldas, Jamnadas, Dwarka Das, B W Wadia
R R Karandikar, Col. J.C. Wedgwood. Later on, Pt. Jawahar Lal Nehru, Netaji Subhash Chandra Bose,
VV Giri, Sarojini Naidu, C R Das and several of other political leaders of the freedom struggle were
associated with subsequent conferences and work of AITUC giving impetus to the work.
 AITUC was formed to give India representation at the International Labour Organization (ILO) of the
League of Nations. Hence statement 2 is correct.
 Mahatma Gandhi did not participate in the first session of AITUC. He had reservations about the
organisational form of trade unions. He regarded the formation of AITUC as hasty and premature. Hence
the statement 3 is not correct.
 AITUC in its second session in 1921 in Jharia had adopted a resolution of Swaraj (Complete
independence from British rule), almost eight years before the platform of freedom struggle- the Indian
National Congress adopted such resolution in 1929.
 In the aftermath of the Second World War, the AITUC played a significant role in the foundation of
World Federation of Trade Unions (WFTU), in the conference held in London. AITUC was represented
by S.A. Dange, R.A. Khedgikar and Sudhindra Pramanik. This conference adopted workers charter.
 The spit in AITUC occurred in 1929 at its 10th session in Nagpur.
 The factors that influenced the formation of AITUC:
o Class consciousness and class solidarity
o Industrial unrest aroused by grave economic difficulties created by First World War
o Russian revolution of 1917
o Establishment of the International Labour Organisation in 1919.

Q 95.C
 Article 14 of the Constitution of India enshrines Equality before Law and Equal Protection of
Laws. However, the rule of equality before law is not absolute and there are constitutional and other
exceptions to it. These are mentioned below:
 The President of India and the Governor of States enjoy the following immunities (Article 361):
o The President or the Governor is not answerable to any court for the exercise and performance of the
powers and duties of his office.
o No criminal proceedings shall be instituted or continued against the President or the Governor
in any court during his term of office. Hence statement 1 is correct.
o No process for the arrest or imprisonment of the President or the Governor shall be issued from
any court during his term of office. Hence statement 2 is correct.
o No civil proceedings against the President or the Governor shall be instituted during his term of office
in any court in respect of any act done by him in his personal capacity, whether before or after
he entered upon his office, until the expiration of two months next after notice has been delivered to
him.
 No person shall be liable to any civil or criminal proceedings in any court in respect of the publication in a
newspaper (or by radio or television) of a substantially true report of any proceedings of either House
of Parliament or either House of the Legislature of a State (Article 361-A).
 No member of Parliament shall be liable to any proceedings in any court in respect of anything said or any
vote given by him in Parliament or any committee thereof (Article 105).
 No member of the Legislature of a state shall be liable to any proceedings in any court in respect of
anything said or any vote given by him in the Legislature or any committee thereof (Article 194).
 Article 31-C is an exception to Article 14. It provides that the laws made by the state for implementing
the Directive Principles contained in clause (b) or clause (c) of Article 39 cannot be challenged on the
ground that they are violative of Article 14. The Supreme Court held that ―where Article 31-C comes in,
Article 14 goes out‖.
 The foreign sovereigns (rulers), ambassadors and diplomats enjoy immunity from criminal and civil
proceedings. The UNO and its agencies enjoy the diplomatic immunity.

Q 96.B
 The council of ministers consists of three categories of ministers, namely, cabinet ministers, ministers of
state, and deputy ministers.
 The cabinet is a smaller body consisting of 15 to 20 ministers. The cabinet ministers head the important
ministries of the Central government like home, defence, finance, external affairs and so forth.
 The Cabinet meets, as a body, frequently to deliberate and take decisions regarding the transaction of
government business. It directs the council of ministers by taking policy decisions which are binding
42 www.visionias.in ©Vision IAS
on all ministers. It supervises the implementation of its decisions by the council of ministers. Hence,
statement 3 is correct.
 Role of Cabinet:
o It is the highest decision-making authority in our politico-administrative system.
o It is the chief policy formulating body of the Central government.
o It is the supreme executive authority of the Central government. Hence, statement 2 is correct.
o It is an advisory body to the president and its advice is binding on him.
o It is the chief crisis manager and thus deals with all emergency situations.
 The word Cabinet was inserted in Article 352 of the Constitution in 1978 by the 44th Constitutional
Amendment Act. Article 352 only defines the cabinet saying that it is ‗the council consisting of the
prime minister and other ministers of cabinet rank appointed under Article 75. Hence, statement 1
is not correct.

Q 97.C
Embryo Splitting
 The term embryo splitting (or "embryo twinning") refers to the formation of twins or multiples through
the artificial microsurgical splitting of an embryo at the cleavage or blastocyst stage. Hence, statement 1
is correct.
 At the earliest stage of embryogenesis, the cleavage stage, the single cells of the embryo (blastomeres) are
still totipotent. This property is presupposed in the blastomere biopsy procedure, where single blastomeres
are removed from the embryo, which under suitable conditions continue to develop like undivided
embryos. The technique of blastocyst bisection involves the separation of the embryo in the later
blastocyst stage into two equal halves. After separation, the genetically identical embryos can continue to
develop.
 In view of the long-term aim of research cloning – the production of autologous stem cells for therapeutic
purposes – the techniques of embryo splitting currently play a minor role. Embryo splitting allows it to
obtain ideally several embryos with identical genetic features from one embryo. Regarding the derivation
of autologous stem cells for therapeutic purposes, it is however not the number of embryos, but the
creation of embryos that share identical genetic features with potential patients that matters. It is from
these embryos that autologous stem cells can be derived in the next step.
 In the context of assisted reproduction treatments, the procedures of embryo splitting potentially provide
the opportunity to gain a higher number of embryos by dividing one embryo, which increases the chance
of successful nidation and therefore the overall likelihood of a pregnancy. Hence, statement 2 is correct.
 Whereas these procedures have commonly and successfully been applied in livestock breeding, they have
not been applied in human medicine as a result of legal restrictions and the more complex development
processes in human cells. With regard to the (potentially) successful application of the procedure on
primates and humans, research groups come to different conclusions, especially in connection with the at
times reduced development capacity of 'divided' embryos.
 In the context of basic research, the procedures of embryo splitting facilitate standardized studies because
the embryos produced this way are genetically completely identical. Studies in this context are currently
being conducted in veterinary contexts. If and to what extent these procedures and objectives can be
realized in human medicine is presently the subject of discussion.

Q 98.A
 The Constitution empowers the Parliament to make laws on any matter enumerated in the State List under
the following five extraordinary circumstances:
 When Rajya Sabha Passes a Resolution
o If the Rajya Sabha declares that it is necessary in the national interest that Parliament should make
laws with respect to a matter in the State List, then the Parliament becomes competent to make laws
on that matter.
o Such a resolution must be supported by two-thirds of the members present and voting. The resolution
remains in force for one year; it can be renewed any number of times but not exceeding one year at a
time.
 During a National Emergency
o The Parliament acquires the power to legislate with respect to matters in the State List, while a
proclamation of national emergency is in operation.

43 www.visionias.in ©Vision IAS


o The laws become inoperative on the expiration of six months after the emergency has ceased to
operate.
 When States Make a Request
o When the legislatures of two or more states (a single state cannot request the Parliament to
make a law on a state subject.) pass resolutions requesting the Parliament to enact laws on a
matter in the State List, then the Parliament can make laws for regulating that matter.
o A law so enacted applies only to those states which have passed the resolutions. However, any other
state may adopt it afterwards by passing a resolution to that effect in its legislature. Such a law can be
amended or repealed only by the Parliament and not by the legislatures of the concerned states.
 To Implement International Agreements
o The Parliament can make laws on any matter in the State List for implementing the
international treaties, agreements or conventions. This provision enables the Central government
to fulfil its international obligations and commitment.
o Some examples of laws enacted under the above provision are United Nations (Privileges and
Immunities) Act, 1947; legislations relating to environment and TRIPS.
 During President‘s Rule
o When the President‘s rule is imposed in a state, the Parliament becomes empowered to make
laws with respect to any matter in the State List in relation to that state. A law made so by the
Parliament continues to be operative even after the president‘s rule.
 Hence, option (a) is the correct answer.

Q 99.D
 Mahatma Gandhi Employment Guarantee Act 2005 (or, MGNREGA, later renamed as the "Mahatma
Gandhi National Rural Employment Guarantee Act" or MGNREGA), is labour law and social
security measure that aims to guarantee the 'right to work'. This act was passed in September 2005.
 It aims to enhance livelihood security in rural areas by providing at least 100 days of wage
employment in a financial year to every rural household whose adult members volunteer to do unskilled
manual work. The MGNREGA scheme contains a provision for an expansion of the scheme to allow for
150 days of work per household for districts affected by drought or other natural disasters.
 The MGNREGA was initiated with the objective of "enhancing livelihood security in rural areas by
providing at least 100 days of guaranteed wage employment in a financial year, to every household
whose adult members volunteer to do unskilled manual work".
 Another aim of MGNREGA is to create durable assets (such as roads, canals, ponds, and wells).
 Employment is to be provided within 5 km of an applicant's residence, and minimum wages are to be
paid. If work is not provided within 15 days of applying, applicants are entitled to an unemployment
allowance. That is, if the government fails to provide employment, it has to provide certain unemployment
allowances to those people. Thus, employment under MGNREGA is a legal entitlement.
 MGNREGA is to be implemented mainly by gram panchayats (GPs). The involvement of contractors is
banned.
 Apart from providing economic security and creating rural assets, NREGA can help in protecting the
environment, empowering rural women, reducing rural-urban migration and fostering social equity,
among others."
 Hence option (d) is the correct answer.

Q 100.A
 Kaziranga National Park and Tiger Reserve
o Kaziranga National park is home to more than 2200 Indian one-horned rhinoceros,
approximately 2/3rd of their total world population. In the year 1985, the park was declared as
a World Heritage Site by UNESCO.
o The park area is circumscribed by the Brahmaputra River, which forms the northern and
eastern boundaries, and the Mora Diphlu, which forms the southern boundary. Other notable
rivers within the park are the Diphlu and Mora Dhansiri. Hence statement 1 is not correct.
o Also, the park is recognized as an Important Bird Area by BirdLife International for the
conservation of avifaunal species. Birds like lesser white-fronted goose, ferruginous duck, Baer‘s
pochard duck and lesser adjutant, greater adjutant, black-necked stork, and Asian Openbill stork
specially migrate from the Central Asia during the winter season.

44 www.visionias.in
 Flora:
o Due to the difference in altitude between the eastern and western areas of the park, here one can see
mainly four types of vegetation‘ like alluvial inundated grasslands, alluvial savanna woodlands,
tropical moist mixed deciduous forests, and tropical semi-evergreen forests. Kumbhi, Indian
gooseberry, the cotton tree, and elephant Apple are amongst the famous trees that can be seen in the
park.
 Fauna:
o Kaziranga has the largest population of the Wild water buffalo anywhere accounting for
about 57% of the world population. Kaziranga is also home to 9 of the 14 species of primates found
in the Indian subcontinent. Hence statement 2 is correct.
o The forest region of Kaziranga Park is home to world‘s largest population of Indian Rhinoceros.
Other animals that can be seen in the elephant grass, marshland and dense tropical moist broadleaf
forests of Kaziranga are Hoolock Gibbon, Tiger, Leopard, Indian Elephant, Sloth Bear, Wild water
buffalo, swamp deer, etc. ‗Kazi 106F‘, country‘s only Golden Tiger resides here. Hence statement 3
is correct.
o Kaziranga has flat expanses of fertile, alluvial soil formed by erosion and silt deposition by the
Brahmaputra. The landscape consists of exposed sandbars, riverine flood-formed lakes known
as, beels, (which make up 5% of the surface area), and elevated regions known as, chapories, which
provide retreats and shelter for animals during floods.
o One-horned rhino:
 It is included in the Schedule – I of Wildlife (Protection) Act, 1972.
 Status : Vulnerable
 Population : 3,500+ individuals ( In northeastern India and the Terai grasslands of Nepa)
 Habitats : Tropical and Subtropical Grasslands, Savannas, and Shrublands
o Wild Buffalo (Bubalus arnee)
 The wild buffalo is mainly found in the alluvial grasslands, marshes, swamps and river valleys.
 The estimated population of the wild buffaloes in the Northeast is around 3,000-4,000, the largest
in the country and accounting for 92% of the world population.
 It is included in the Schedule 1 of the Wild Life (Protection) Act, 1972.
 IUCN Status : Endangered

Copyright © by Vision IAS


All rights are reserved. No part of this document may be reproduced, stored in a retrieval system or transmitted
in any form or by any means, electronic, mechanical, photocopying, recording or otherwise, without prior
permission of Vision IAS.

45 www.visionias.in ©Vision IAS

You might also like